Paper B e-EQE 2021: The jury is still out

This year’s paper B was very difficult. The client's wishes and proposed amendments could scarcely be reconciled with the disclosure of the application as filed. Even among DP's experienced attorneys and tutors, this year’s paper led to dramatically different solutions.

Although it was promised that "Paper B will have the same syllabus and character as before", Paper B looked different in design from previous years. This year’s candidates were confronted, for the first time, with third-party observations, a computer-implemented invention (or not?), and client's claims with unusual (and incomplete) tracking of changes. In particular, the accumulation of these issues appears to have placed a very high burden on the candidates.

So far, we have been able to draw up several "solutions", each of which has its strengths and shortcomings - both in the light of meeting the wishes of the client, as well as good defensibility against the requirements of Art. 123(2), and with a clear narrative for the problem-solution approach.

For that reason, we will present below, for the first time, several proposals for amended claims, and our considerations thereof. 

Claim 1 could have been amended as follows:

1. A container (1) for organic refuse, comprising

a. an upper compartment (3) having a support (3a) for holding the refuse (7) populated with earthworms (8);

b. a lower compartment (4) for collecting excess moisture (6), wherein optionally the lower compartment (4) is removable;

c. drain holes (5) permitting moisture to drain from the upper to the lower compartment, or when the lower compartment (4) is removable directly into soil if the upper compartment (3) is placed directly on the soil; and

d. an upwardly opening covering lid (2) which fits over the container (1), wherein the container comprises a water spraying device (13) connected to a water container (14).


  • The client had removed "for organic refuse", but there only seems to be support for a container for "organic (kitchen) refuse"
  • The client had added "and/or housefly eggs"; while there is basis for (earth)worms and optional additives (cf. [022]), which may be  housefly eggs (cf. [003]), there seems to be no basis for "housefly eggs" alone as implied by the "or" language; the "housefly eggs" as additives can be put in a dependent claim
  • The client had deleted the "lower compartment" and the "drain holes", because, according to the client, "the lower compartment and the drain holes are both optional and are not essential to the container
    • deleting both the lower compartment and the drain holes, without adding a replacement feature, seems to certainly contravene Art. 123(2) as the invention clearly requires excess water to be drained. A possible solution is to replace both features by a more functional definition, e.g., “means for draining moisture from the refuse”, with possible support from [020] combined with [004], but ultimately we expected it not to be directly and unambiguously derivable from the application as filed, i.e., to contravene Art. 123(2) (and it would be very much against the established style of Paper B in which the wording of amendments are directly derivable from the wording of the application as filed - including for functional claim language)
    • the drain holes seem essential, because they not only are a means for controlling the moisture of the refuse, but they also prevent the earthworms from falling from the upper compartment into the lower compartment (and the soil) (cf. [017])
    • the lower compartment is trickier: both embodiments described and in the drawings have the lower compartment. But [018] says it may be removable; does this mean that it is non-essential and can be deleted? Indeed, we can argue that it was not explicitly explained as essential, is not indispensable for the invention (it can e.g. also drain on soil) and no compensating features are required for its removal - but is a container with only an "upper compartment" directly and unambiguously derivable from the application as filed as being an invention per se? 
    • The present amendment "wherein optionally the lower compartment (4) is removable" is our safe bet (and not completely in line with the client's wishes) - but we feel that a claim without the lower compartment and a convincing Art. 123(2) argumentation should get (more?) marks, too
  • The client had added "a water spraying device (13) for adjusting the moisture" and argued that this created novelty over D2 as "in D2 the water spraying device is exclusively used for cleaning" That argument does not hold, since the spraying device of D2 - attached to a tap - is of course also suitable for adjusting the moisture. [012] mentions a water spraying device (13) connected to a water container (14)
    • it is easy to see that the amendment "connected to a water container" creates novelty over the tap of D2. It also allows arguing a convincing PSA starting from D1 as CPA: a skilled person seeking to improve moisture control in the refuse, would not consider D2 as it is concerned with removing water, and it merely teaches to use a spraying device connected to a tap for cleaning the container
    • however, we can also understand that candidates struggled with adding the "water container" part to the claim, as this seems to be an external component to the container (see Fig. 2), and therefore seems to modify the claim in a "kit-of-parts". We also struggled with that, and with the fact that it results in a claim that is easy to circumvent.

An alternative solution could be:

1. A container (1) for organic refuse, comprising

a. an upper compartment (3) having a support (3a) for holding the refuse (7) populated with earthworms (8);

b. a removable lower compartment (4) for collecting excess moisture (6);

c. drain holes (5) permitting moisture to drain from the upper to the lower compartment, or directly into soil if the upper compartment (3) is placed directly on the soil; and

d. an upwardly opening covering lid (2) which fits over the container (1), wherein the container comprises a water spraying device (13)  for spraying water on the refuse


  • Novel over D1 (no spraying device at all) and over D2 (no removable lower compartment) and similar PSA story, with the addition that a removable lower compartment is undesirable w.r.t. larvae being present there
  • But: rather limited w.r.t. to the original claim, and also not fully in line with client's wishes.
  • In this solution, the connection to the water container is removed, but could also be included (see above for arguments; if you conclude that the water container as such is not indispensible for the function it serves, which is to supply water to the water spraying device, as it could be any type of water source/supply)

Further possible amendments (possibly as alternative solutions to limiting claim 1 to "connected to water container" and/or "removable") were:

  • a moisture detector (with a display)
    • In D1, but not in D2
    • present in all embodiments that use a water spraying device
    • seems required for the moisture data step of the calculation method
    • not suggested in any way by client, so rather arbitrary (not B style) amendment
  • adjustable size of the drain holes (cf. [017])
    • it contributes to the invention's effect of active control of the moisture
    • however, again arbitrary in view of the client's wishes

As far as the other (independent) claims are concerned:


The claim to a method for producing a fertilizer (original claim 4) was amended by the client to include

  • "and/or housefly eggs"; as for claim 1, not supported
  • "earthworms" instead of "worms"; this amendment (supported by [006]) is indeed required to overcome the Examiner's objection 
  • a backreference to the preceding container claims; that makes sense for restoring novelty over D1 and D2
  • specifying "by spraying water on said refuse" (NB: not tracked, and not explained by the client!) in step b


The following claim seems to be an acceptable amendment, also taking into account the removable lower compartment/soil option of amended claim 1:

X. A method for producing a fertilizer, the method comprising the following steps

a. providing a container (1) according to any of claims 1-xx, comprising organic refuse (7) and earthworms (8);

b. optionally, adjusting the moisture of the refuse by spraying water on the refuse and/or by draining moisture from the refuse;

c. composting the refuse with the earthworms for a time sufficient to convert the refuse into fertilizer, and,

d. optionally, separating earthworms from the fertilizer.

  • the step "and/or by draining moisture from the refuse" is likely not required because it can also be achieved automatically by the container. Novelty is already conferred by the use of the container of the invention itself


Finally, again  - in our view - two viable options for dealing with "mental act" claim 6:

9. A computer-implemented calculation method for optimizing processing organic refuse, comprising the steps of

a. defining a target value TV at time point tp > 0 for the amount of the earthworms (8) present in the refuse (7) at said time point tp;

b. receiving data relating to the moisture of refuse (7) and the amount of earthworms (8) at a plurality of time points tp;

c. determining if the value for the amount of earthworms (8) present in the refuse (7) at tp > 0 is equal to the defined target value TV for the amount of earthworms (8); and

d. recommending adjusting the amount of moisture added to the refuse (7) at said time point tp, if the value for the amount of earthworms (8) present in the refuse at tp > 0 is not equal to the defined target value TV for the amount of earthworms (8).

 

  • Basis in [024]: "One, more or all steps of the method can be implemented by a computer"
  • GL G-II 3.5.1: "Where a claim defining a method for performing mental acts as such is limited by specifying that the method is carried out by a computer, not only the use of a computer but also the steps carried out by the computer themselves may make a technical contribution if they are based on technical considerations and serve a technical purpose"
  • The main argument against this claim is that a large majority of EQE candidates is not familiar with computer-implemented inventions; on the other hand, any EQE candidiate is expected to be familiar with the main concepts of any technical field at a base level when sitting the exam, and you do not need to be more familiar CII than what the Guidelines and G 3/08 tell to be able to amend this claim as proposed above. In that sense, CII is not different from, e.g., second medical use.

 

However, based on the proposal of the client, the following claim should also be allowable:

X. A calculation method for optimizing processing organic refuse, comprising the steps of

a. providing a container according to any one of claims 1 to xx comprising organic refuse and earthworms,

ba. defining a target value TV at time point tp > 0 for the amount of the earthworms (8) present in the refuse (7) at said time point tp;

cb. receiving data relating to the moisture of refuse (7) and the amount of earthworms (8) at a plurality of time points tp;

dc. determining if the value for the amount of earthworms (8) present in the refuse (7) at tp > 0 is equal to the defined target value TV for the amount of earthworms (8); and

ed. recommending adjusting the amount of moisture added to the refuse (7) at said time point tp, if the value for the amount of earthworms (8) present in the refuse at tp > 0 is not equal to the defined target value TV for the amount of earthworms (8).

 

  • It should be specified that the container comprises organic refuse and earthworms as the container of claim 1 itself does not necessarily has them present
  • With the container in the claim, the claim can no longer be objected to under Art.52(2)&(3)
  • GL G-II 3.5.1: "A claimed method is not a method for performing mental acts as such if it requires the use of technical means (e.g. a computer, a measuring device, etc.) to carry out at least one of its steps or if it provides a physical entity as the resulting product"
  • With the container with the earthworms and the organic refuse in the claim, also the earthworms and the organic refuse are no longer abstract concepts but real things with technical character, so that also the further method steps are clearly technical and well-linked.
  • Main argument against this claim is that the "steps can be executed by a computer" information is not used.

With respect to the third party observations, we note that:
  • The announcement was published on the internet under www.opendayincotsford.com on 6 June 2018 does not disclose any container
  • Even though there were over 100 visitors who were impressed by the guided tour on 02 August 2018 at 11:45, where the public had access to the facilities and where the apparatus of claim 1 was located, the public could not see the internals of the container, not could they see the drain holes at the bottom of the container. There is no indication that the public could hold or open the containers to inspect them. As these claimed features were not visible, the open day on 02 Aug 2018 does not impact novelty of inventive step of the claim.
  • Further, there are no details given as to how the public could "learn how our earthworms and house flies produce fertilizer and feed", nor any evidence.

We did not want to delay posting a reasonable effort; however, because we feel we still may have overlooked something, we may post an updated answer in a few day's time. We will also provide at a later date (more of) our findings, including the dependent claims.

We look forward to your comments!
Comments are welcome in any official EPO language, not just English. So, comments in German and French are also very welcome!

Please do not post your comments anonymously - use a nickname (or your real name) instead - it makes responding more difficult and rather clumsy ("Dear Mr/Mrs/Ms Anonymous of 02-03-2021 22:23"), whereas using your real name or a nickname is more personal, more interesting and makes a more attractive conversation. You do not need to log in or make an account - it is OK to just put your (nick) name at the end of your post. Thanks!

The DeltaPatents Team


Note: you may also wish to check the comments posted to our first impressions blog, here.

Comments

  1. I am the above biologist. For claims 4 and 6 I introduced references to claims 1 to 3, whereby the same arguments for novelty and inventive step would also apply. I think for claim 6 it is even essential to introduce a technical step (a techincal purpose and/or a techincal step, e.g. a reference to claims 1 to 3). In this regard, I dont think simply saying it is a CII can overcome the objections. A techicnal purpose or step is needed, but not simply re-wording the claim.

    ReplyDelete
  2. @BS: I have a different opinion. You have to make a choice and only if two documents are equivalenr starting points in terms of technical field and purpose, you do PSA starting from different documents.

    This comment has put it in a nutshell: Anonymous March 08, 2021 12:03 pm

    ReplyDelete
  3. In the method claim 4, why would you also make step b not optional and require the spraying of the water? Doesn't the inclusion of the novel and inventive container provide the same reasons for the method claim to be novel and inventive? And surely requiring spraying of water makes the claim commercially unappealing as there will be times when performing the method that the moisture is fine and therefore does not need water adding - which this will then not be covered by the claim. It does say in the application that moisture does not often need to be added, it is far more common that moisture needs to be removed.
    (ps I did actually miss that the amended claims added the spray feature given the EPO forgot to track it, so didn't really give too much consideration to adding the spray, but thinking about it now, I actually still stand by it is not needed for the reasons above).

    ReplyDelete
  4. In my opinion, adding "removable" for the lower compartment seems to narrow the scope of the protection? What if a competitor just has two independnet products: one is the same product but the lower compartment is not removable, and another one without the lower compartment at all..

    ReplyDelete
  5. In my opinion, adding "removable" for the lower compartment seems to narrow the scope of the protection? What if a competitor just has two independnet products: one is the same product but the lower compartment is fixed and not removable, and another one without the lower compartment at all...

    ReplyDelete
  6. @BS: I see your point, that it appears to be weird that the claim is inventive even though with D2 as CPA it would not be, but this is exactly the case here. At least in mechanics, it is very often decisive which document is CPA, so we argue very carefully with the examiner on this point.

    Longstanding BoA case law, in my opinion, justifies the view that the CPA election is an integral and indispensable part of the PSA. (Otherwise, there would not be such a strong focus on it in the eqe in Part C either). The exam board wants us to internalize the principle of CPA and apply it correctly!

    This also applies to the towel VS bed sheet exapmle. The bed sheed will never be CPA.

    Please don't confuse "purpose" as a criterion for CPA with the claim interpretation for novelty, where "suitable for" is fairly enough.

    I think the below comment of "Anonymous", March 08, 2021 11:03 pm puts this in a nice nutshell.

    ReplyDelete
  7. I think this all makes sense, although the container probably needs to be included with the water spray to satisfy A123 as these are never disclosed separately.

    Although I am not sure that claim alternatives to holes need to be disclosed in the two figures to satisfy A123(2) EPC. The test is just whether the feature is directly and unambiguously derivable from the application as filed, which I reckon this functional definition meets for the reasons I gave above (i.e. drain holes perform the function of draining excess moisture, and there are other known ways of draining excess moisture).

    ReplyDelete
  8. In the feature "recommending adjusting the amount of moisture added" the term adjusting is generic and includes lowering moisture. Still can't see a general problems linking claim 6 with the remaining method. Further, the term recommending implies that info are shown on display, but does not imply any adjustment as per the first independent method claim. It seems more that this can be objected at inventive step for lack of technical charater as related to display of information. Would not be just better to remove claim 6?

    ReplyDelete
  9. In my opinion, adding "removable" for the lower compartment seems to narrow the scope of the protection? What if a competitor just has two independnet products: one is the same product but the lower compartment is fixed and not removable, and another one without the lower compartment at all...

    ReplyDelete
  10. @BS: I see your point, that it appears to be weird that the claim is inventive even though with D2 as CPA it would not be, but this is exactly the case here. At least in mechanics, it is very often decisive which document is CPA, so we argue very carefully with the examiner on this point.

    Longstanding BoA case law, in my opinion, justifies the view that the CPA election is an integral and indispensable part of the PSA. (Otherwise, there would not be such a strong focus on it in the eqe in Part C either). The exam board wants us to internalize the principle of CPA and apply it correctly!

    This also applies to the towel VS bed sheet exapmle. The bed sheed will never be CPA.

    Please don't confuse "purpose" as a criterion for CPA with the claim interpretation for novelty, where "suitable for" is fairly enough.

    I think the below comment of "Anonymous", March 08, 2021 11:03 pm puts this in a nice nutshell.

    ReplyDelete
  11. Now I think about it, I think the amendment to "means for" that you have done makes a lot of sense and is in line with previous EM papers like you say - clever and wish I thought of it myself!!!

    ReplyDelete
  12. @BS: I see your point, that it appears to be weird that the claim is inventive even though with D2 as CPA it would not be, but this is exactly the case here. At least in mechanics, it is very often decisive which document is CPA, so we argue very carefully with the examiner on this point.

    Longstanding BoA case law, in my opinion, justifies the view that the CPA election is an integral and indispensable part of the PSA. (Otherwise, there would not be such a strong focus on it in the eqe in Part C either). The exam board wants us to internalize the principle of CPA and apply it correctly!

    This also applies to the towel VS bed sheet exapmle. The bed sheed will never be CPA.

    Please don't confuse "purpose" as a criterion for CPA with the claim interpretation for novelty, where "suitable for" is fairly enough.

    I think the below comment of "Anonymous", March 08, 2021 11:03 pm puts this in a nice nutshell.

    ReplyDelete
  13. @BS: I see your point, that it appears to be weird that the claim is inventive even though with D2 as CPA it would not be, but this is exactly the case here. At least in mechanics, it is very often decisive which document is CPA, so we argue very carefully with the examiner on this point.

    Longstanding BoA case law, in my opinion, justifies the view that the CPA election is an integral and indispensable part of the PSA. (Otherwise, there would not be such a strong focus on it in the eqe in Part C either). The exam board wants us to internalize the principle of CPA and apply it correctly!

    This also applies to the towel VS bed sheet exapmle. The bed sheed will never be CPA.

    Please don't confuse "purpose" as a criterion for CPA with the claim interpretation for novelty, where "suitable for" is fairly enough.

    I think the below comment of "Anonymous", March 08, 2021 11:03 pm puts this in a nice nutshell.

    ReplyDelete
  14. @BS: I see your point, that it appears to be weird that the claim is inventive even though with D2 as CPA it would not be, but this is exactly the case here. At least in mechanics, it is very often decisive which document is CPA, so we argue very carefully with the examiner on this point.

    Longstanding BoA case law, in my opinion, justifies the view that the CPA election is an integral and indispensable part of the PSA. (Otherwise, there would not be such a strong focus on it in the eqe in Part C either). The exam board wants us to internalize the principle of CPA and apply it correctly!

    This also applies to the towel VS bed sheet exapmle. The bed sheed will never be CPA.

    Please don't confuse "purpose" as a criterion for CPA with the claim interpretation for novelty, where "suitable for" is fairly enough.

    I think the below comment of "Anonymous", March 08, 2021 11:03 pm puts this in a nice nutshell.

    ReplyDelete
  15. I totally agree re the functional language for the drain hole, I removed them completely and now regretting it...

    ReplyDelete
  16. removable lower compartmentMarch 08, 2021 12:48 pm

    In my opinion, adding "removable" for the lower compartment seems to narrow the scope of the protection? What if a competitor just has two independnet products: one is the same product but the lower compartment is fixed and not removable, and another one without the lower compartment at all...

    ReplyDelete
  17. @BS: I see your point, that it appears to be weird that the claim is inventive even though with D2 as CPA it would not be, but this is exactly the case here. At least in mechanics, it is very often decisive which document is CPA, so we argue very carefully with the examiner on this point.

    Longstanding BoA case law, in my opinion, justifies the view that the CPA election is an integral and indispensable part of the PSA. (Otherwise, there would not be such a strong focus on it in the eqe in Part C either). The exam board wants us to internalize the principle of CPA and apply it correctly!

    This also applies to the towel VS bed sheet exapmle. The bed sheed will never be CPA.

    Please don't confuse "purpose" as a criterion for CPA with the claim interpretation for novelty, where "suitable for" is fairly enough.

    I think the below comment of "Anonymous", March 08, 2021 11:03 pm puts this in a nice nutshell.

    ReplyDelete
  18. I don't think that the term "water-absorbent" should be used in claim 1, as this term would surely raise a clarity objection.

    ReplyDelete
  19. Again, i am the above biologist. Honestly, i do not think introducing a "removable" under compartment is a good solution. It even further restrict the claim actually. The cleint intructred to have something broader, and now you make it even worse than the original one. I removed the under compartment like the client instructed, but maintained the drain holes, because the drain holes are essential for the invention.
    Regarding claims 4 and 6, I introduced reference signs to any one of claims 1 to 3, whereby the same arguments can apply to claims 4 and 6.
    In respect of claim 6, I think it is essential to introduce a techincal purpose and/or technical step (e.g. introducing references to any one of claims 1 to 3). Although i do not have many experiences in this regard, i heard a lot of seminars. Simply saying it is CII would not overcome the objections, and a technical feature is essentially needed.

    ReplyDelete
  20. Same here, I made that amendment too - since it seemed essential
    also, it was very useful to be novel over all prior art docs

    but even if i did it, I also have the impression it was:
    - probably a 'result to be achieved' - like feature
    - probably not sufficiently disclosed

    ReplyDelete
  21. removable lower compartmentMarch 08, 2021 12:56 pm

    In my opinion, adding "removable" for the lower compartment seems to narrow the scope of the protection unnecessarily? What if a competitor just has two independnet products: one is the same product but the lower compartment is fixed and not removable, and another one without the lower compartment at all...

    ReplyDelete
  22. @ AnonymousMarch 08, 2021 12:03 pm

    You talk about CPA, but first, we have to discuss novelty!

    Adding the water spraying device does not give novelty over D2, even if D2 would be in a different field (suitable for!).

    ReplyDelete
  23. Certanly it lacks unity and technical character without an active step indicating that that water is adjusted by means of....
    A method for "recommending adjusting the amount of moisture added to the refuse" just displays a warning that moisture should be adjusted. This lacks technical character in that is a mere display of information without any further use. Further the term adjusting is generic in that it does not exclude reducing moisture. In contrast, spraying devices only add water. I questioned the unity and for that I suggested a divisional and removed claim 6. I could not spend much more time on it.

    ReplyDelete
  24. Hi BS,
    Great question and I doubt I have a satisfactory answer but let me try. You say, "So even if it is not CPA, let's just assume it is, only for the purpose of analyzing IS with respect to D2."
    So if we assume that it is, which it can't be, but let's say it is: then I would say all things being equal/same, non-transparent lid is not inventive over D2.
    But this is assuming all things being equal. But all things are not equal. We have the drain holes as well in this amended claim because without them we can't proceed. Regardless of novelty/inventive step, drain holes or some kind of mechanism for excess moisture removal has to be there, as DP points out correctly.
    So then we have (at least) two distinguishing features over D2 (if D2 is unnaturally taken as CPA) and both of them help in providing the adequate moist and dark environment in which earthworms thrive. The objective problem may be formulated as: Providing an optimum environment for earthworms to flourish while composting efficiently.

    That's the best I have got...

    ReplyDelete
  25. BS I replied to you but guess it disappeared. So now I will write more concisely.
    If D2 is unnaturally taken as the CPA then D2 will have two missing features: nontransparent lid and drain holes for moisture removal. Drain holes have to be there regardless of novelty/inventive step as some kind of mechanism for moisture removal has to be provided as DP correctly points out.

    So the effect of these is to provide an optimum dark and moist environment for earthworms. So the objective problem may be formulated as: "Providing an optimum environment for earthworms to flourish while compositing efficiently."

    Sorry that my previous answer got lost, which was slightly more expansive .

    ReplyDelete
  26. @BS:in every opposition proceeding I have taken part to date, the board required us to always make a clear and well-grounded selection of the CPA. Only when we could make a valid argument why two or more documents are equally good starting points, they allowed more than one discussion of the inventive step. And D1 and D2 in no way are equally good starting points.

    Hence, I would be surprised if D2 was allowed as CPA or as a starting point for the discussion of the inventive step.

    ReplyDelete
  27. Replied to you twice now. Both times it disappeared...

    ReplyDelete
  28. Spraying device connected to a water containerMarch 08, 2021 1:32 pm

    In my opinion, one just needs to add the feature "the container comprises a water spraying device connected to a water container" in order to make claim 1 new and inentive over D2:

    1. D2 discloses a water tap, not a water container -> claim 1 is new over D2.

    2. D2 belongs to a different field; a skilled person has no motivation to consult D2. Further, D2 even "teaches away" from adding water, since (i)D2 aims to "remove water or moisture" from chicken manure (see title, line 10 of D2) and (ii)the disclosed spray device is used for cleaning but not for adding moisture. In view of D2, a skilled person thus "would" never arrive at a container for organic refuse, comprising a water spraying device for spraying water on the refuse.

    ReplyDelete
    Replies
    1. At first I made the same Argument...then I dreaded the 0 points for non-inventiveness and added the non-transparent lid to be in the safe side.

      But you could be correct.

      Delete
    2. Correction: I dreaded the 0 points for lack of novelty over D2

      Delete
    3. Isn't a water tap novel over a water container? a tap cannot hold water...

      I also had similar arguments on spraying device connected to a water container, but I'm not sure if that's enough to make the examiner happy. I find it a bit ridiculous that we still do not agree at this point what is the real solution the examiner is looking for -- considering we are all sort of experts on patent... (aren't we?)

      Delete
    4. Correction: I mean a water container seems to be new over a water tap, as a water tap cannot hold water.

      Delete
  29. BS I replied to you but guess it disappeared. So now I will write more concisely.
    If D2 is unnaturally taken as the CPA then D2 will have two missing features: nontransparent lid and drain holes for moisture removal. Drain holes have to be there regardless of novelty/inventive step as some kind of mechanism for moisture removal has to be provided as DP correctly points out.

    So the effect of these is to provide an optimum dark and moist environment for earthworms. So the objective problem may be formulated as: "Providing an optimum environment for earthworms to flourish while compositing efficiently."

    Sorry that my previous answer got lost.

    ReplyDelete
  30. Not sure but, do you think the "EVAPORATION" is a sign that you do not need a specific means for draining excess moisture ? Without drain holes or absorbent material, you can just allow it to evaporate?

    ReplyDelete
  31. @JP: all three answers are there and they are great (and in my eyes correct). I guess when you read this, you found the "load more" button :)

    ReplyDelete
  32. @BS: I see your point, that it appears to be weird that the claim is inventive even though with D2 as CPA it would not be, but this is exactly the case here. At least in mechanics, it is very often decisive which document is CPA, so we argue very carefully with the examiner on this point.

    Longstanding BoA case law, in my opinion, justifies the view that the CPA election is an integral and indispensable part of the PSA. (Otherwise, there would not be such a strong focus on it in the eqe in Part C either). The exam board wants us to internalize the principle of CPA and apply it correctly!

    This also applies to the towel VS bed sheet exapmle. The bed sheed will never be CPA.

    Please don't confuse "purpose" as a criterion for CPA with the claim interpretation for novelty, where "suitable for" is fairly enough.

    I think the below comment of "Anonymous", March 08, 2021 11:03 pm puts this in a nice nutshell.

    ReplyDelete
  33. @BS: I see your point, that it appears to be weird that the claim is inventive even though with D2 as CPA it would not be, but this is exactly the case here. At least in mechanics, it is very often decisive which document is CPA, so we argue very carefully with the examiner on this point.

    Longstanding BoA case law, in my opinion, justifies the view that the CPA election is an integral and indispensable part of the PSA. (Otherwise, there would not be such a strong focus on it in the eqe in Part C either). The exam board wants us to internalize the principle of CPA and apply it correctly!

    This also applies to the towel VS bed sheet exapmle. The bed sheed will never be CPA.

    Please don't confuse "purpose" as a criterion for CPA with the claim interpretation for novelty, where "suitable for" is fairly enough.

    I think the below comment of "Anonymous", March 08, 2021 11:03 pm puts this in a nice nutshell.

    ReplyDelete
  34. When something can simply be done by evaporation, what holes we are talking about ? Isn't it clear that they are not essential?

    ReplyDelete
    Replies
    1. The drain holes themselves are not essential, but if you leave them out, or make them optional/an alternative, you have to add something to the claim which is removing the excess moisture from the container. The application gave the following alternative options: water-absorbing article and evaporation. But the evaporation is not very well described. At some point I was also pondering about the question whether the evaporation approach would actually work with the container as shown in the figures. You could argue that it would, e.g. if the container lid is opened at a regular basis or just remains opened; you have to make sure that the water vapour trapped in the container can efficiently diffuse out of it. But this is a bit counter-intuitive in respect of the remainder of the client's application, i.e. avoid that you perceive malodour or that rats and the like could get access to the refuse. It would also make the lid unnecessary and seems to lead nowhere, at least when it comes to getting marks for answering the paper.

      Delete
  35. Oh no, I was sure the extra lid with moisture retaining surface (can't remember the whole thing) was the only inventive feature. The problem, I thought, was retaining moisture, which was proven by the last document (R&D paper Moisture v Substrate). Nothing else looked in anyway inventive. Back to the drawing board and life in general I suppose.

    ReplyDelete
  36. @Thomas : Unity was the biggest issue. I spent a lot of time pondering how to stay board while avoiding to fall foul of R43(2) and/or A82. The CII method alone could be made novel and inventive with some technical means (even broadly claimed). However, how can you reconcile this with the container from claim 1 or the method from claim 4? The method of claim 4 is ok if it is claimed as a method of operating the container of claim 1. But the CII method of claim 6 would need to be made dependent on claim 1 too. The basis for that was a bit sketchy though. Maybe the best was to add a separate note telling the applicant to file a divisional for that one. But that would have been another 30minutes down the drain .... I cannot get past the fact that this paper was not fully vetted and for some reason constituted the emergency backup (read draft 2022) of the original paper intended for us (which must have been compromised in its secrecy close enough to the actual exam to leave no time for any checks)

    ReplyDelete
  37. I'm almost 100% with this approach, except that I think the moisture detector with display should also be added, so that it becomes unitary with claim 6. Simply stating the method is implemented in a computer without any further technical features could still be a mental act?

    ReplyDelete
  38. Am I the only one, who deleted the drain holes and the lower compartment from claim 1? IMO it fulfills the gold standard:
    1) Par.[019] states that to produce the organic refuse, all you need is a (generic) container filled with organic waste and earthworms. The step of controlling the moisture is described as optional in p. [019] under b.
    2) paragraph [020] states "the moisture of the refuse can be adjusted by spraying water on the refuse and/or by draining moisture from the refuse"
    -> a plain container with only the water spraying device can be implicitly derived from [019] and [020]
    3) P. [004] further substantiates that the skilled person knew that there are plenty different ways to reduce moisture (drain holes, absorbant, vaporization)on the application date.
    4) the lower compartment is also obviously optional, in view of p.[018]

    ReplyDelete
    Replies
    1. You're not the only one Simon, exactly my way of thinking as well as arguing!

      Delete
    2. @Simon - I did the same. Although I had reasoning, I also followed the DeltaPatents guidance that the client's amendments are mostly/nearly always novel/inventive except that there could be added matter/intermediate generalisation issues hence I included the water container to address these issues. Although I didn't get to fully argue my inventive step etc. but this was my reasoning for going with those deletions plus with basis I found for doing so.

      Delete
    3. @Simon
      I'm adraid - deletion of the drain holes isnot in line with [19].
      [19] relates to the method. By contrast, [7] states that "The purpose of the present invention is to provide a container, which allows the control of moisture in organic refuse in the container."

      Delete
  39. I think a lot of people's points in the comments are valid, as are the arguments in support of the DP solutions.

    Personally I went through a lot of these points when amending the claims. This, together with the fact there were three independent claims, and extensive formatting issues, meant it took me two hours to amend the claims I didn't finsih Inventive Step.

    I've seen a lot of comments saying that marks will probably be awarded for multiple solutions, but what about those of us who didn't finish? I don't know what the EPO is going to do but it would be incredibly unfair and disheartening to accept multiple solutions but not consider the fact that the exam required a lot longer than 3.5 hours to do properly...

    ReplyDelete
    Replies
    1. In view of the Examiner's reports from the previous years, they assign a lot of points for IS arguments. It's like amendments 25-30pts, basis for amendments 20pts, Novelty 10pts, IS 40pts, others (clarity/unity/etc.) 10pts... but there are three independent claims this year, those 40pts should be distributed and the importance of a single claim should be diluted.

      Delete
  40. Dear DP - thank you for your solutions. I am glad that many experienced DP tutors found this paper difficult (not in a bad way) but it shows that under pressure of 3.5 hours - this paper was not doable for candidates. I had to skip some of the amendments section to get to other sections and deal with other issues such as 3rd party observations.

    Based on previous papers - 70 marks were allocated to argumentations so this is why I decided to sacrifice the amendment section (30 marks) and move on. I would feel that it would be completely unfair if they decide to change the marking now because this would clearly change outcomes and it is not in line with previous papers.

    ReplyDelete
    Replies
    1. It shows that this year's paper B was not doable. It was impossible to do a decent job of every section. Just NOT good.

      Delete
    2. I agree with you anon - but I went the other way of focussing on the amendment because I assume you get more marks in the argumentation section if you have 'the right' amendment.

      But obviously this meant I ran out of time on inventive step! So there is no easy way of marking the paper. Do you get more marks for at least getting to the end or do I get marks for getting the amendment but not finishing? IMO they can't mark this terrible excuse of a D paper without being 'completely unfair'. Best thing to do would be to award everyone a baseline of 20 marks, and go from there. (of course they won't do this)

      Delete
    3. @ A Slow Typer - like you, I ran out of time for inventive step due to not being sure of the right amendment which, with all things considered above, obviously took a long time to digest to try and figure out the right direction - not that there is a right direction from the looks of it.
      Hoping they change the allocation of markings this year otherwise it won't be looking good for me from the lack of/no IS arguments.

      Delete
  41. I wonder why "moisture detector (11) with a display (12)" could be omitted in independent claim 1.

    It seems that "water spraying device (13) connected to a water container (14)" should be introduced into claim 1, but the "water spraying device (13) connected to a water container (14)" is only disclosed together with "moisture detector (11) with a display (12)" in the container in para [12].

    [012] In Fig. 2, the container (1) has a standard moisture detector (11) with a
    display (12) and also a water spraying device (13) connected to a water container (14).

    Even though, para [14] discloses:

    [014] Preferably, the compost container has a standard moisture detector (11) with a display (12).

    Is this meaning that "moisture detector (11) with a display (12)" is optional in the container which is already described to have "a standard moisture detector (11) with a display (12) and also a water spraying device (13) connected to a water container (14)"?

    ReplyDelete
    Replies
    1. It can be omitted because it was such a bad paper - there are plenty of alternatives.

      Delete
  42. BS I replied to you but guess it disappeared. So now I will write more concisely.
    If D2 is unnaturally taken as the CPA then D2 will have two missing features: nontransparent lid and drain holes for moisture removal. Drain holes have to be there regardless of novelty/inventive step as some kind of mechanism for moisture removal has to be provided as DP correctly points out.

    So the effect of these is to provide an optimum dark and moist environment for earthworms. So the objective problem may be formulated as: "Providing an optimum environment for earthworms to flourish while compositing efficiently."

    Sorry that my previous answer got lost.

    ReplyDelete
  43. BS I replied to you earlier about the transparent lid but guess I think that thread has reached its limit and its not publishing new comments there anymore. Anyway, before replying to that, I see your claim above, and I have to agree that this is probably the model claim with one or two minor additions (such as earthworms or earthworms and eggs can be easily added. not and/or but simply A or A+B. but no biggie I guess. secondly I would stick with drain holes and let water absorbent material alone, i.e., not mention it). Anyway, overall (with the above caveats) your answer is the one in my view.
    So you don't have to read below as I already agree with you. But because you had asked, here's the answer:

    If D2 is unnaturally taken as the CPA then D2 will have two missing features: nontransparent lid and drain holes for moisture removal. Drain holes have to be there regardless of novelty/inventive step as some kind of mechanism for moisture removal has to be provided as DP correctly points out.

    So the effect of these is to provide an optimum dark and moist environment for earthworms. So the objective problem may be formulated as: "Providing an optimum environment for earthworms to flourish while compositing efficiently."

    Anyway, your answer is better and I would probably lose marks for the unnecessary limitation for the non-transparent lid.

    ReplyDelete
    Replies
    1. Hi Johan,
      do not forget that the Examiner equates the mesh floor of D2 to drain holes for moisture removal (although I still struggle with the question whether mositure removal should be only from the refuse or the container entirely; a non-removable lower compartment as in D3 is a source and a sink of moisture).
      This whole discussion on the CPA does not quite meet reality. Reading the GL for the determination of the CPA, it is made clear that there is no solid basis for an absolute interpretation of the quality of being closest:

      "In the event of refusal or revocation, it is sufficient to show on the basis of one relevant piece of prior art that the claimed subject-matter lacks an inventive step: there is no need to discuss which document is "closest" to the invention; the only relevant question is whether the document used is a feasible starting point for assessing inventive step (see T 967/97, T 558/00, T 21/08, T 308/09 and T 1289/09). This is valid even if the problem identified in a problem-solution reasoning may be different from the one identified by the applicant/patentee.

      As a consequence the applicant or proprietor cannot refute the argument that the claimed subject-matter lacks inventive step by submitting that a more promising springboard is available: a piece of prior art on the basis of which the claimed invention is considered non-obvious cannot be "closer" than a document on the basis of which the claimed invention appears obvious, because it is evident in this situation that the former does not represent the most promising springboard from which to arrive at the invention (T 1742/12, Reasons 6.5; T 824/05, Reasons 6.2)."

      On the other hand, CPA is used as a criterion to avoid a multiplicity of IS attacks by the opponent (also GL for determination of the IS). What can we conclude from this? Any document can be CPA if it leads to a successful IS attack, but to reduce the effort of examining each possible IS attack based on a different selection of the CPA you have to present convincing arguments as to why you think your choice of the CPA is worth closer examination.

      Delete
  44. I also focused on the two independent claims with the backreference first. I left it in and argued as DP above.
    Since the client had expressly stated that the narrow scope of his amendments was sufficient, I just accepted it. The client‘s letter provided a further hint in that the Client considered the backreference to resolve the A.52(2)(c) objection, which is true.

    ReplyDelete
  45. Me too. There was a sentence somewhere saying that the container allowed to simultaneously fertilize and produce eathworms or something like that. A mesh in D3 does not necessarily have to be tightly knit. D2 lets the larvae go through. The drain holes are essential, the removable container is not, according to both, the client’s wishes and the description. The claim with the spray, the water container (always disclosed together) and the holes configured to not let the earthworms go through is new and inventive over all references.
    I did not find support for the container of claims 1-3 and the method. That is added SM. The few reference numbers were insufficient to support the combination with the claims. Agree with DP amendment to the algorithm claim.
    Very difficult paper indeed. I got stuck on many issues in claim 1. Did not have time to even finish one IS argument. The earthworm/worm/housefly egg larvae and pupae debacle was very muddy as well. Tough to let a client down with additional experimental results. But I got stuck from the get go with the client’s wish to delete “for organic refuse”.
    I also missed that I had left in an optionally. Not having strikethrough made it very hard to keep track of the markings in the claims. It is a bit comforting to know you struggled with this paper as well.

    ReplyDelete
  46. Real talk: If the client suggests and is happy with back-referencing both method claims to claim 1, in a paper B that is already loaded with information and very short on time, why would anyone not keep these amendments, and then necessitate an entire additional problem-solution-approach?

    ReplyDelete
    Replies
    1. That's the tricky part of B.. Client can suggest something which makes the protection scope narrower or even agaist EPC. if we, as patent attorneys, spot a better/broader solution then we should write down our solution!

      Correction: we are not attorneys yet until we pass EQE :(

      Delete
    2. "What was emphasised a couple of times is that it is very important to follow the client’s instructions. First, as was the case with the previous single papers B, the points for the claims are coupled to the amendments proposed by the client. Second, the comments and suggestions of the client are really intended to assist the candidates to find the solution. The client is the authority in terms of the commercial products to be protected. Rule 24 IPREE was cited for emphasising that it is the client who instructs about the way to proceed with the application: “it is about amending the claims of the client, according to the clients instructions letter, in order to meet the requirements of the EPC”.

      taken from Tutors‘ Report on the EQE 2019 Papers in 04/2019

      In view of the above, including the back references is what the client, i.e. the authority, wants, and thus should in principle be followed

      Delete
    3. I didn't (and still don't) think there was basis for the back reference to all of the container claims in the method claims. Yes, we should do what the client wants, IF it is allowable under the EPC. The client is always going to suggest something that isn't allowable for added matter reasons as that is part of the test. You get zero marks for claims for just submitting the client's draft claims in their entirety. I would have kept these amendments in if I thought there was clear basis for them to save a lot of work, but I couldn't convince myself this was the case. Unfortunately this year it wasn't at all clear what should be done instead!

      Delete
    4. If you take the German language instructions from the client literally then, by making the claims 4 and 6 dependent, you contradict his instructions because he did not want ANY new dependent claims:

      [008] Bitte nehmen Sie an diesem Anspruchssatz alle Änderungen vor, die Sie für nötig
      erachten, damit die Ansprüche die Erfordernisse des EPÜ erfüllen, ohne weitere
      abhängige Ansprüche aufzunehmen.

      In the English version it is a bit more clear because there the client refers to his amendments in that statement. I know that this is a bit far fetched but it is Just another contradictory statement in this exam.

      Delete
  47. I agree, I do not believe they wanted to see a second problem solution approach (but I am not sure). In 2019, they wanted two psa but in 2019 a TPO was missing and the claim amendments were more straightforward.

    ReplyDelete
    Replies
    1. They were not: independent claims in the Examiners' Report of 2019 are clearly non-unitary, i.e. do not satisfy EPC. It is not possible to find them if you search for a unitary solution.

      Delete
  48. Hi, although I rather follow ostrich tactics when feeling bad about exams, however considering the underperformance of the B-committee EPO I herewith provide my input. Mind you I did not check the other comments.

    First of all I found it impossible to get a good grasp considering we had to scroll through the document. There was no good way for comparing the invention, the claims, the proposed claim amendments and basis for these amendments. Side-by side comparisons were not possible and reformatting the assignment in the text editor is too cumbersome. Although the scrolling function was a bit better for the additional tab (since it was not a page by page scrolling), for me this extra tab was not very useful. I copied the claims to my text editor and put small comments to it, however this limited me in the sense that for this tab the assignment had to be scrolled page by page and no search function was present. This scrolling cost me a lot of time (I guess 15-30 minutes). Worse it added to the confusion since there were a lot of claims and a lot of very strange things which needed an uncluttered view of the facts (see also all the other comments in this blog). What aggravated the situation is that the amendments were annotated poorly (bold??) and incomplete and figure reference numbers were missing (such numbers are important for me as a Chemist to understand).

    What bothers me is that the EPO did not keep its end of the bargain. The deal is that we candidates prepare thoroughly, whilst the EPO provides for a challenging but predictable exam within boundaries set beforehand and the EPO provides for a suitable exam environment. The EPO clearly erred in both factors but in the end the candidates will pay for that. The aim is to test if we are fit for practice, but now what will happen is that marks will be stretched, some compensation may occur to reach an ‘acceptable’ pass rate. However, even with ‘compensation’ it will be a raffle who will pass and who not (considering the confusing nature of the exam and the multiple possible solutions(, so it really will not be the people fit to practice will pass but just the lucky ones. Because of that I think the EPO should have everyone passed.

    Furthermore, although the EPO continuously communicates the E-EQE is learning together (candidates and EPO) no such thing happened. There were warnings beforehand, that time for B was too short, highlighting function is very relevant etc. but the EPO did not demonstrate a learning attitude (flexible) but just went ahead without any reflection on the feedback and on the exam there were planning to provide.

    ReplyDelete
    Replies
    1. totally agree. The technical issues regaridng formatting, tabs, lack of printing/highlighting meant it was not possible to give this exam a good go in 3.5 hours. It would have been ok if, say, there was only one dependent claim, no TPO, and a shorter application. However, INCREASING the content compared to previous papers whilst expecting us to work within the ridiculous costraints of the new system was basically impossible.

      Also agree that who passes will be a lottery. I ran out of time, perhaps those who rushed to finished will be favoured if multiple solutions are accepted.

      Delete
  49. While I agree that both solutions offered by dp in order to solve the objection under Art.52(2)&(3) for claim 6, the claim still lacks unity of invention and technical character. In particular, feature
    “d. recommending adjusting the amount of moisture added to the refuse (7) at said time point tp”
    is just displaying information regarding the container status and without any functional use of this information displayed the claim is simply void of a further technical effect.
    In essence, without the active step of “adjusting the moisture of the refuse by spraying water on the refuse” [based on the displayed information] there is no unity here.
    For this reason, it would be better to remove claim 6 from the set.
    Anyway very tough paper….

    ReplyDelete
  50. My solution:

    I started by considering the client wanted ‘and/or fly eggs’ which was abundantly clear from the letter, but the whole invention was about worms. I found that incredibly strange, considering the whole thing was written for worms. I saw the para at the end on ‘additives’ and at the beginning on prior art additives but only could come to an ‘and’ claim. I thought this would not be in the interest of the client considering the limiting nature of such ‘and’ claim whilst a ‘preferably’ does not add and only limits the scope of protection. Here my mind froze considering the client clear wish for fly eggs whilst adding such eggs does not help the client. I parked the issue for later but unfortunately did not have time to address.

    I left the ‘for organic refuse’ in considering this is only a ‘suitable for use’ and I saw no real limitation in this. I really found such amendment trivial and not worth considering. I know this is not what the EQE demands, but questioning such trivialities in my eyes is really beside the point and I resent such utter trivialities in the exam.

    I though the lower compartment could be removed as not being essential but the drain holes not. For I while I considered to further dimension the size of the holes in order to be novel over D2 (prevent earthworms from falling from para 17) but in the end decided to ‘sail close to the wind’ also for inventive step and to be novel only for the water container (that cannot be un-intermediate generalized with the spraying device). The latter added to my annoyance; who in real life would write such an application and specify a water container instead of a general source of water (or instead of no source at all), and the same is valid for D2 (only with a tap), there is really no reason except for the sake of the exam. Really an artificial situation.

    Inventive step starts with D1 as CPA, and humidity controlling as OTP, and I still wonder if based on the intention of D2 of lowering water content one would not apply the spray of D2 (as it is advertised for cleaning).
    The display of claim 3 I though should remain in, and the lower compartment as a dependent claim seemed a good idea.

    As I was really time-pressured I took the escape the client offered me to have claim 5 depend on the previous claims and therewith providing novelty and inventive step. The worm clarity issue I found again trivial but taking up a lot of thinking time. Considering the invention is fully on earthworms and even the letter of the client is on earthworms and there is basis; in real life I would not have hesitated one second to amend.
    I do understand if the claims are clear there is no need for amendments and ‘worms’ is clear (any worm). However something else is written in the description which makes the description rather than the claims unclear. What to do with this situation? I kept to common sense and amended to earthworms.
    I had no time for any further deep thoughts for example to consider the optional features in combination with the previous claims.

    The same applied for claim 5, no time to consider, if the client wants this claim out I will take it out.

    ReplyDelete
  51. For claim 6 I only had 5 minutes left. I am a chemist and not into computer-implemented. I though a mental act in a box is still a mental act, and I saw some basis to add a computer and an output device and without much thinking added these features to the claim. I had no time for novelty or inventive step or consider other options.

    The public disclosure I found time-consuming (time-line, assessing what was demonstrated, understanding the invention and comparing what was demonstrated). I think the B-committee was wrong here to present a third party observation in this way; according to OJ 2017 A86 the EPO must give its opinion and comment on its relevance. The Examiner of this B exam failed to do so.
    I had no time to quote guidelines or case law. On the content side: in the end it was unclear what was demonstrated and therefore not relevant in my opinion.

    So all in all, I found this an artificial exam poorly touching upon reality and way too long and not suitable for the E-EQE platform.

    ReplyDelete
    Replies
    1. I agree, there should have at least been a slight hint in the client letter as to where to go with the TPO, since a lot of people I have spoken to spent a lot of time over this and even thought about novelty/IS over this disclosure. A simple line in the client letter like "we don't think this is at all relevant since the visitors could not even see inside the container" would have been enough to save the majority of candidates a lot of concern and time spent.

      Delete
  52. It seems to me that it is risky to argue that according to the invention the water spraying device is connected to a water container and not to a water tap, and that a connection to a water tap is not a specific way of a connection to a water container. There are two major drawbacks:
    - some kind of controllable flow device, such as a water tap, a valve, a pump, etc., between the spraying device and the container shall be considered to be implicitly disclosed in the original application documents. Otherwise spraying water on demand would not have been disclosed in an enabling way. Unfortunately there is no such explicit disclosure in the original application documents. Bringing forward the fact that no tap is explicitly disclosed or claimed in favour of novelty is therefore risky, one might run into article 83 or 100(b) issues.
    - the future proprietor might in real life loose protection when the spraying device is connected to a water container, such as a water pipe, with a tap, and potentially all equivalents. Indeed if the above argument is brought forward by the applicant in EP proceedings, it can be used against a proprietor in national proceedings. That’s most certainly not in the interest of the applicant, because it would make the patent useless.

    I hope that the papers are not meant as a purely abstract exercise, where it just has to be assumed that a connection to a water container excludes a connection to a water tap, only because different expressions are used and there is no information in the paper pointing to the contrary. If that would be the case, how should a candidate know where to stop? D2 discloses a spray cleaning device, not a water spraying device. If these two devices should be considered to be different, because different expressions are used and the paper doesn’t define that they are corresponding in terms of technical features, then the connection to the water container would anyway not be necessary to achieve novelty over D2.

    ReplyDelete
    Replies
    1. In 2019, it was necessary to restrict the melting point of the salt in claim 2 to 130-350°C to achieve novelty (over D3 if I recall correctly). Thus, the scope of protection could also easily be circumvented by selling heat storage unit of claim 2 with a salt having a melting point below 130°C (easy work-around for competitors).

      I dont think the eqe is too much concerned with how potential competitors could circumvent the scope of protection so I wouldnt overthink it in the exam. I believe "connected to a water container" was the solution to establish novelty over D2, even if it may have potential drawbacks with regard to scope of the claim in infringement proceedings.

      Delete
    2. I think that drafting allowable claims, that provide a protection as close as possible to the wishes of the applicant, and being able to argue that the requirements of the EPC are met, is the main point of paper B. The applicant for sure doesn’t want to have protection for a container without the possibility to start and stop the spraying of water, and to loose protection for a container which has that feature. This would go completely against the spirit of the application.
      This might seem far fetched, but similar things happen. In a real life case dealing with videogame controllers, an attorney in EP proceedings had argued that strain gauges in a prior art document do not anticipate force sensors as claimed. In the context of the application, this argument is rather absurd, and based only on completely abstract reasoning. However, it was decided in national proceedings that using sensing elements equivalent to strain gauges is therefore no patent infringement. As all force sensors in such controllers are based on measuring deformation or strain and derive force therefrom, the patent had become useless by argumentation in the EP phase.
      The answer to a question such as whether or not strain gauges are specific force sensors, should of course not be based on the technical knowledge of an EQE candidate. It should be derivable from the information given in the paper. The answer to a question such as whether or not a connection to a water tap is a specific form of a connection to a water container, should be given based on experience from daily life, when it is not derivable from the information provided in the paper. When using the spray device in D2’s container, meaning that the water tap is open, a connection to something which contains water, so a water container, is established. This should not be considered as undisclosed in D2, even in the context of an EQE paper.

      Delete
  53. In 2019, it was necessary to restrict the melting point of the salt in claim 2 to 130-350°C to achieve novelty (over D3 if I recall correctly). Thus, the scope of protection could also easily be circumvented by selling heat storage unit of claim 2 with a salt having a melting point below 130°C (easy work-around for competitors).

    I dont think the eqe is too much concerned with how potential competitors could circumvent the scope of protection so I wouldnt overthink it in the exam. I believe "connected to a water container" was the solution to establish novelty over D2, even if it may have potential drawbacks with regard to scope of the claim in infringement proceedings.

    ReplyDelete
  54. Concerned CandidateMarch 10, 2021 11:03 am

    I could have sworn there was basis for removal of the lower compartment and the drains - wondering how much of an issue this can be. From the sounds of it, a complete failure. Curious as to how many others made such an amendment whilst including the connected to a container.

    ReplyDelete
  55. Just have a question about the feature of "a water spraying device for adjusting the moisture" which is a functional feature, wherein the function is "adjusting the moisture (especially adding the moisture)". Can we say it is novel over the "spray cleaning device" in D2? Because in D2 it is necessary to reduce the moisture (over 50%), and "spray cleaning device" is no way used to or suitable for adding the moisture in D2. In addition, due to the different functions, their structures may be also different, wherein one is designed for adjusting the moisture, such as with more covering area to efficiently distribute water, and the other is designed for cleaning, which provides more concentrated and more powerful spray to clean (for example, hydrant and tap are both outlets of water but with totally different functions and structures).

    ReplyDelete
    Replies
    1. I would say that you are right in all practical circumstances, where the structural features of the cleaning device would surely be different from the ones of a moisture-adding device. In practise, you would most likely be informed on these structural differences and you can then rely on them.
      This is different however from the very artificial situation of the B paper, where you are not told anything about the structure. Although your way of thinking is correct, in the framework of the B paper it is a pure speculation. I think that in the absence of any detailed information you are supposed to assume that the cleaning device is still satisfactory for fulfilling the function of adding moisture (e.g. the unknown examiner which issued the B paper communication could argue that the water tap as a valve is only opened slightly so that no high-pressure water jet is obtained or that the water tap is only opened at short intervals, sufficiently short to avoid removal of the organic refuse; ultimately, it will also depend on the thickness of the organic refuse layer). With this in mind, I don't think it is safe in the B paper to rely on this difference in wording in order to establish novelty (remember, the examiners generally use a very broad interpretation of the terms in a claim when assessing novelty).

      Delete
  56. “[007] The purpose of the invention is to provide a container, which allows the control of moisture in organic refuse in the container”

    “[009]A container according to the invention requires minimum skill in its operation and little operator involvement in the composting process.”

    How do we acheive control of the moisture which requires little skill and involvement?

    => By implementing drain holes (5), moisture detector (11) and a water spraying device (13). The moisture detector (11) is a preferred embodiment according to [014]. The client suggested including the water spraying device (13). All three features (5), (11) and (13) are shown in Figure 2.

    If so, arguing IS should be fairly smooth because D1 only mentions moisture monitoring and not moisture control. D2 concerns method for removing water from chicken manure and discloses a spray cleaning device.

    ReplyDelete
  57. A water tab is a water container because it is configured to contain water. Period.

    The amount of water "contained" is irrelevant.

    ReplyDelete
  58. There is absolutely no basis for "providing a container (1) according to any of claims 1-x" in the method claim.

    Indeed, there is a sentense in the description say "the device of Fig.1 can be used in A method for producing earthworms". However, it does not refer to the specific method of the invention. Moreover, Fig.1 refers only to the embodiment wihtout the spraying device. And now claim 1 refers to the spraying device so there is clearly no basis at all. Moreover, it was not necessary in order to be nvoel and inventive. It was novel over D2 because earthworms were used and it was novel over D1 because of the water spraying step. It was inventive over D1+D2 for basically the same reasons as for claim 1 (no incentive to actually spray the worms because in D2 it was used for cleaning). Finally it was in line with the client wishes to protect also other other ways of regulating the water (evaporation etc.) because this method claim would not have been limited to the draining holes, to the contrary of claim 1 in which it was not possible to remove the draining holes for A123(2) reasons.

    ReplyDelete
    Replies
    1. Mind Willing to UnderstandMarch 12, 2021 1:42 pm

      The skilled person interpreting the application with a Mind Willing to Understand would consider the question "how the containers disclosed with various optional features and references to Figs. 1 and 2 would be operated?". Naturally, the skilled person would consider the invented method as presenting the answer to this question, not only because of the link through paragraph [0010], but also because the container and the method are presented as parts of one invention. So, Art. 123(2) is satisfied.

      As for the client instructions, indeed the client did not want independent claims to be limited to drain holes. However, the client did not want the method to be limited by spraying too: he consistently kept this feature optional.

      Delete
  59. Case law of the boards of appeal:
    "In T 916/15, the board took the view that the jurisprudence of the boards of appeal referring to "a mind willing to understand" did not apply for the purpose of assessing the allowability of amendments under Art. 123(2) EPC. It followed from T 190/99 that this concept applied only where it was necessary to interpret a claim of a granted patent for the purposes of Art. 123(3) EPC and Art. 69 EPC."

    So it seems that the application of "the mind willing to understand" is not to be applied for the assessment of compliance to A123(2).

    Generally speaking, we have to apply the gold standard which is: is the combination of features directly and unambiguously disclosed ? the answer is no in view of my previous message.

    Moreover, I do not remember even once that the concept of "mind willing to understand" was applied in paper B.

    ReplyDelete
    Replies
    1. Sorry, my message was in response to "Mind Willing to UnderstandMarch 12, 2021 1:42 pm"

      Delete
    2. Mind Willing to UnderstandMarch 12, 2021 2:50 pm

      What is the purpose of your cherry-picking of one exceptional case from II-E, 1.3.9? It starts with "In T 1946/10 the board held that according to established jurisprudence the skilled person interprets a claim with a mind willing to understand ...".

      Also, you ignored the following argument: "The skilled person interpreting the application ... would consider the question "how the containers disclosed with various optional features and references to Figs. 1 and 2 would be operated?". Naturally, the skilled person would consider the invented method as presenting the answer to this question, not only because of the link through paragraph [0010], but also because the container and the method are presented as parts of one invention. So, Art. 123(2) is satisfied".

      Delete
    3. I am not cherry-picking. Firstly, we are talking about a paper B not a real life case. The basic question to answer for compliance to A123(2) is not "how the containers disclosed with various optional features and references to Figs. 1 and 2 would be operated?" but is rather "Is the combination of the specific method disclosed directly and unambiguously in combination with the specific embodiment of Fig.2 (comprising the water spraying device)". The answer is No, as explained in my previous post.

      Moreover, there is clearly conflicting case law regarding the application ofthe "mind willing to understand" for compliance to A123(2), therefore it is very doubtful that they expected an amendment based on this justification.

      Delete
    4. Mind Willing to UnderstandMarch 12, 2021 5:32 pm

      Well, if you want to pretend that you do not understand that the industrial applicability of the containers disclosed with various optional features and references to Figs. 1 and 2 lies in the invented method, then let be it.

      Indeed, these containers may be just for cuteness. I hope that you are happy now.

      Delete
    5. Can we agree that it is at the very least ambiguous?

      Delete
    6. Mind Willing to UnderstandMarch 12, 2021 9:58 pm

      I cannot subscribe to this view, at the very least yet.

      According to GL F-II, 4.9, "the description should indicate explicitly the way in which the invention is capable of exploitation in industry, if this is not obvious from the description or from the nature of the invention"; and "in most cases, the way in which the invention can be exploited in industry will be self-evident, so that no more explicit description on this point will be required".

      So, what do you think: are the invented containers industrially applicable, or not? If yes, what Method, satisfying F-II, 4.9, for their exploitation is indicated in the description or is self-evident from it?

      Delete
    7. I agree with "Mind willing to understand" that there is good argument that there is disclosure for limiting the method by the container.

      However, I also agree with Rolp that there is argument to the contrary.

      I do not think that the argument with respect to industrial applicability has any merit, since there is no indication in the case-law or guidelines or anywhere that some "self-evident" or "obvious" method of industrial applications is clearly and unambigiously disclosed in the application. It is simply two things.

      In the passage of GL F-II 4.9 cited above, even the term obvious is used. ("should indicate [...], if this is not obvious [...]"). Additionally, it is "should", not "must". There is no lack of industrial applicability, merely because the way of industrial applicability is not shown in the description.

      Some way of operating the container may be obvious from the description. However, subject-matter, which is merely obvious to the skilled person cannot be taken as clearly and unambigiously disclosed. The standard for disclosure is far stricter.

      There clearly is no explicit disclosure of providing a container with the features of amended claim 1 in the method of claim 4/5. The respective paragraphs of the application are simply silent on this.
      Any disclosure allowing for the combination of the method and product claims could only be an implicit one. An implicit disclosure must not be confused with subject-matter being obvious. As the GL states in H-IV 2.2: "However, the question of what may be rendered obvious by that disclosure in the light of common general knowledge is not relevant to the assessment of what is implicitly disclosed by that document".

      It may be obvious that the container should be used with the method. This obviousness may be enough for showing industrial applicability of the container (in fact, it is enough that the container can be manufactured and be put on the market, it is completely irrelevant how the container can or should be used for Art. 57). No conclusion can be drawn from this for Art. 123(2) EPC.

      If at all, the connection between method and container is implicit in the application. As "mind willing to understand" correctly points out, if the application is taken as a whole, the skilled person could probably understand nothin else than that the container is to employed in the method. However, Rolp does have a point that this is far from certain.

      In any case, I agree with Rolp that the advantage of deleting the reference to the container from claim 5 is that the protection will be broader. Spraying with water is what makes the invention inventive over D1. Adding the container to the method will limit the method to the water sprayer anyways. Thus, the limitation with water spraying in the method claim is not really relevant. However, the method without reference to the container will convey protection for different types of drying, which is exactly what the client wants.

      Delete
    8. If as you say it is far from certain, that means that it is not Directly and unambiguously disclosed. Simple.

      Delete
    9. Mind Willing to UnderstandMarch 13, 2021 2:11 pm

      @AnonymousMarch 13, 2021 12:04 am

      You wrote quite much about legal standards, but still there is the issue of how the invented containers can be used, according to the application and the relevant standard. If there is no method to use them, then these claims have to be deleted to satisfy EPC.


      Regarding "the advantage of deleting the reference to the container from claim 5 is that the protection will be broader", this is incorrect. Rather, the protection will be in a different direction.

      When claim is limited by water spraying, it will be infringed only in specific circumstances (dry and hot weather).

      In contrast, when the method claim is limited by the container features, it will be used all the time, even when water just drains through the holes, but only if the specific container is used.

      What limitation is more severe, we do not know. The client did not give instructions regarding the method, except for the proposed claim amendment, which kept water spraying as merely an option. According to the client, drain holes is also a non-essential option, for the container.

      Is it better to cover the method limited by spraying or the method limited by containers? More likely yes, but this depends on whom the client is ready to sue, on the weather in about next 20 years, on how inessential the holes are.

      Delete
    10. (Im anonymous from above)

      @Mind Willing to Undestand

      You seem to assume that there must be some disclosure of a method for using the container in order to meet the requirements of Art. 57 EPC? Then you make the assumption that the application is in accordance with Art. 57 EPC and based on this assumption and your aforementioned "rule" you assume that the application must disclose a method of use of the container and this method can only be the method of claim 4.

      I still do not subscribe to this reasoning. First of all, if your argument that Art. 57 EPC requires the disclosure of some use method would be correct, it would really say nothing about the disclosure of such a method. It could also mean that the claim of the container violates Art. 57 EPC. Furthermore, I think your assumption is incorrect. Art. 57 EPC does not require the disclosure of a method of using the container. As I have said before, for the purposes of Art. 57 EPC it is absolutey enough that the container is a physical entity that can be manufactured and sold. Also claim 1 provides for some use of the container ("for holding organic refuse"). That is also certainly enough to fulfill Art. 57 EPC.

      So no, if there is no method of using a product disclosed in the application, respective product claims would not have to be deleted to satisfy the EPC. Furthermore, as per your cited GL, obviousness of a method for use is enough. However, obviousness of such a method is definitively not enough for Art. 123(2). I think there simply is no connection between the requirements of Art. 57 EPC regarding the container and Art. 123(2) and the disclosure of using the claimed container in the claimed method. And even following your rule, it is clearly possible that the application sufficiently shows a method of using the container in the sense of Art. 57 (method being obvious), but fails to do so in the sense of Art. 123(2) (obvious is not enough).

      @Rolp

      I did not say that. I said it is far from certain that the method claim as amended by the client is clearly and unambigiously disclosed, i.e., meets the requirements of Art. 123(2) EPC. I think both cases can be argued. There is no rule that when in doubt, Art. 123(2) is not fulfilled.

      I still tend to follow "Mind willing to understand" with respect to Art. 123(2), but not based on his/her argument with respect to industrial applicability, but rather under the concept of implicit disclosure.

      Regarding the scope of protection:

      We do not relay know, how the feature "adjusting the moisture of the refuse by spraying water on said refuse" would be interpreted in infringement proceedings by a national court and whether an infringer would actually have to spray water to infringe the claim. The essence of the step is "adjustment" of moisture and it could be argued that the step, when interpreted as to its technical purpose (what is for example the standard applied by German courts) rather has the meaning of having means for spraying and using said means only to the extent necessary. I think "adjustment" could also mean keeping the moisture unchanged, if it is already satisfactory.

      I think there is good arguments for adding the container and keeping the adjustment of moisture optional. There is also good arguments for not having the container in the claim, but rather making the adjustment of moisture mandatory. After all, the client gets protection for the container under claim 1. That also confers protection for any method employing the protected container. If the method claim is made dependent on the container, its additional scope with respect to claim 1 would be very narrow.

      Delete
    11. There is an interesting chapter in the white book of case law discussing obvious Vs implicit disclosure.I agree that it is obvious that the container if claim 1 can be used in the claimed method. I think in real life, it could be argued that it is an implicit disclosure. However, we have to take the into account: this is paper B, not real life, fig 1 refers to a first embodiment, fig. 2 refers to a second embodiment, the method is another embodiment ( literally written in the description).Combining embodiments is not allowed. Paragraph 10 only says that one of these embodiment can be used in a method for producing X, not the actual claimed method.

      I agree it is a close call. Maybe it just proves one thing: paperB was not well drafted.

      Delete
    12. We can certainly agree on that last half-sentence ;-)

      Delete
  60. Took me 30 mins just to copy, paste and format the clients letter and amended claims into dummflow. So thanks for the several possible solutions clarifying that I failed. Does it really make sence enrolling for 2022 not knowing whether the system will be different?

    ReplyDelete
    Replies
    1. No, not if you still prepare for the paper-style.

      Delete
  61. @BS: I see your point, that it appears to be weird that the claim is inventive even though with D2 as CPA it would not be, but this is exactly the case here. At least in mechanics, it is very often decisive which document is CPA, so we argue very carefully with the examiner on this point.

    Longstanding BoA case law, in my opinion, justifies the view that the CPA election is an integral and indispensable part of the PSA. (Otherwise, there would not be such a strong focus on it in the eqe in Part C either). The exam board wants us to internalize the principle of CPA and apply it correctly!

    This also applies to the towel VS bed sheet exapmle. The bed sheed will never be CPA.

    Please don't confuse "purpose" as a criterion for CPA with the claim interpretation for novelty, where "suitable for" is fairly enough.

    I think the below comment of "Anonymous", March 08, 2021 11:03 pm puts this in a nice nutshell.

    ReplyDelete
  62. @Anonymous 1:51: Explicitly described prior art is part of the application as filed GL G-IV, 5.1.
    I agree with you Anonymous 1:06, basis for "housefly eggs", no basis for "and (earth)worms.

    ReplyDelete
  63. In the real world, I agree that it would not be inventive (maybe, I do not know to be honest, I never dealt with such inventions).

    I thought that in paper B, we have to limit ourselves to what the content of paper B, nowhere was it stated that ""define a value -> receive data (twice) -> calculate another value based on the received data and the defined value". If the data is number of earthworms or anything else, is technically irrelevant. Basically, the examiner would say the programming of an algorithm to output a recommend moisture adjustment value is part of the ordinary daily tasks of a programmer"

    As a non specialist in CII, there is just no way I could have guessed that, could I ? Based on the information given in paper B, the method of claim 4 implemented on a computer, should be inventive no ?

    ReplyDelete
  64. I thought about the same option, including (adjustable) holes preventing the worms from falling.


    However, I did not spot any info in the exam as to how the size of an earthworm compares to the size of a larvae.

    Is a larvae the same size as an earthworm? Or is it much smaller? In the latter case, something which allows for a larvae to travel to the lower compartment might me still small enough, for preventing the rainworm to fall to the lower compartment.

    Under the stress of the exam, I finally decided against the option to go down this rabbit hole. Or, more accurately, to go down this worm hole :-)

    But, maybe you are right.

    Anke

    ReplyDelete
  65. Is there a point in retaking B in 2022 without any indication that the paper will be any less disastrous as in 2021 ?

    ReplyDelete
    Replies
    1. No, not if you ask such a question.

      Delete
  66. The Examiners' Report is available!

    http://documents.epo.org/projects/babylon/eponot.nsf/0/7043D0BB750F0825C12586F8002C8336/$FILE/Compendium_ExRep_2021_B_EN.pdf

    Section 2 of the Examiner's Report shows that the expected set of claims (30 marks) was:

    Claim 1
    A container (1) for organic refuse (7) comprising
    a. a compartment (3) having a support for holding the refuse (7) populated with earthworms (8);
    b. drain holes (5) permitting moisture to drain from said compartment; and
    c. an upwardly opening non-transparent covering lid (2) which fits over the container (1),
    wherein the container (1) comprises a water spraying device (13) for adjusting the moisture.

    Claim 2
    A container according to claim 1, wherein the opening lid (2) is coupled to a floating lid (9) by flexible means (10) such that the floating lid rests on the surface of the refuse (7).

    Claim 3
    A container according to any one of claims 1 and 2, wherein the container has a moisture detector (11) with a display (12).

    Claim 4
    A container according to any one of claims 1 to 3, wherein the container further comprises a lower compartment (4) for collecting excess moisture (6).

    Claim 5
    A method for producing a fertilizer, the method comprising the following steps
    a. providing a container comprising organic refuse (7) and earthworms (8);
    b. adjusting the moisture of the refuse by spraying water on said refuse and by draining moisture from the refuse;
    c. composting the refuse with the earthworms for a time sufficient to convert the refuse into fertilizer, and,
    d. optionally, separating earthworms from the fertilizer.

    Claim 6
    A computer-implemented method for optimizing processing organic refuse, comprising the steps of
    a. defining a target value TV at time point tp > 0 for the amount of earthworms (8) present in the refuse (7) at said time point tp;
    b. receiving data relating to the moisture of the refuse (7) and the amount of earthworms (8) at a plurality of time points tp;
    c. determining if the value for the amount of earthworms (8) present in the refuse (7) at tp > 0 is equal to the defined target value TV for the amount of earthworms (8); and
    d. recommending adjusting the amount of moisture added to the refuse (7) at said time point tp, if the value for the amount of earthworms (8) present in the refuse at tp > 0 is not equal to the defined target value TV for the amount of earthworms (8).

    The Examiner's report indicates how various unnecessary limitations were penalized.

    Some alternative solutions were hardly penalized. "For instance, reference to the unclear feature of preventing earthworms from falling from the upper compartment into the lower compartment in addition to a further distinguishing feature (such as a non-transparent lid or a water spraying device with a container) results in a loss of 2 marks" (end of section 3.1), and "No marks are deducted, when the feature of “by draining moisture from the refuse” is missing or optional" in claim 5 section 3.2).
    W.r.t. the independent method claim claim 6 (5 marks), item 3.3 indicates:
    "It is expected that the method of claim be amended to refer to a computer-implementedm method. This suitably addresses the Article 52(2)(c) EPC objection. No further amendments to claim 6 are expected.
    Adding a technical feature (such as a the container according to claim 1) is more limiting and results in a loss of 2 marks. Keeping claim 6 as proposed by the client also results in a loss of 2 marks".

    70 marks were available for the letter of reply to the EPO.

    The Examiner's Report does not contain any comments as the the side-effects of the online format, nor whether that has somehow been reflected in the marking.

    ReplyDelete
    Replies
    1. The claim 1 answer is wild...

      Delete
  67. "If the container (1) is located in a dark area such as a basement or a shed the covering lid can be transparent'"

    vs

    claim 1 in Examiners' Report: "non-transparent lid"

    Isn't it required to cover all embodiments, as was required before?

    ReplyDelete
  68. there is no pointer in the client's letter at all that such limitation is desired...

    ReplyDelete
  69. Also saying that a water sprayer for adjusting moisture of soil (I had configured to) is not novel over D2's cleaner is a bit of a stretch. We are told to use functional language in the job to get the best for our clients. A cleaner implies cleaning (duh) which arguably would be done without the presence of soil, especially since D2 is all about reducing moisture in the soil. Would definitely be arguable IRL and the soution offered by the EPO would ore likley than not end up with you getting sacked by your client

    ReplyDelete
  70. I agree with Anonymous (June 21, 2021 9:53 am) that a clear pointer to the non-transparent lid as being more important than the other two 'accepted' solutions (moisture detector, adjustable drain holes, both -2 marks) appears to be missing.

    In general, it is ok not to cover all embodiments. As long as you're not going against a clear instruction of the client and no better solution is available, you will have to abandon some embodiment(s). The paper could have been more clear about the client's preferences as to which of the 'acceptable' solutions would have been most desirable.

    Functional language is very useful and often desirable, but you always have to check if the functional language does imply an actual limitation. 'A water sprayer for adjusting moisture of soil' is anticipated by a water sprayer that is provided for cleaning if that water sprayer is 'suitable without modification' for adjusting moisture of soil. The cleaning sprayer is clearly capable of moisturising soil, which makes 'a water sprayer for adjusting moisture of soil' non-novel.

    ReplyDelete
    Replies
    1. So what if 'a water sprayer for adjusting moisture of soil' was adjusted to "a moisture-adjusting water sprayer"?
      In my mind, that is novel and inventive, since D2 mention notion about using the water sprayer for adjusting moisture but only for cleaning. An the moisture-adjusting is now an actual limitation and not only suitable for.

      Delete
    2. a moisture-adjusting water sprayer" aka "a water sprayer for adjusting moisture" is still anticipated by the water sprayer of D2. You need an additional limitation to differentiate from D2 (e.g. detector or non-transparent lid). Although I agree the non-transparent lid makes all more elegant as contributing to the solution, it was not easy to find it.

      Delete
    3. read DP solution - they had attached to water container - that should be enough over D2, but it's not according to the EPO.

      Anyone who amends a claim IRL like the EPO have proposed will be a useless attorney. Clients will not be happy.

      It's almost as if the EPO want attorneys to get used to making hugely limiting amendments to make their jobs easier...

      Delete
  71. Yes, well I actually had the delta patents solution above (word for word aprat from 'configured to' instead of 'suitable for') - so I also had the 'connected to a water container'. Configured to doesn't equal suitable for either...

    I agree with you re ni clear pointers, but in reality the solution offered by the EPO (and the two -2 mark alternatives) are far too limiting. Noted for next year anyway!

    I take comfort in the fact that the Delta Patents solution is the same as mine and also would have resulted in maximum 6/14 marks!

    ReplyDelete
  72. "Same character as before"???

    Not all embodiments covered, multiple correct answers (or with a penalty of only 2 marks) rather than a single solution, no a single fully correct solution so candidates can never feel confident which whatever they answer, ...

    ReplyDelete
  73. 3.2 Independent method claim 5 (9 marks)
    The expected solution for claim 5 comprises deleting the term “optional” in step b) and specifying that the moisture is adjusted by spraying water on the refuse and by draining moisture from the refuse.

    If step b) is missing or optional, 4 marks are lost. A reference to the container of at least independent claim 1 is an unnecessary restriction which leads to a deduction of 3 marks.

    It seems that I have been penalized twice... why should I remove the term "optionally" when including a back reference to claim 1?! Then, the scope is even more restricted. That does not make sense to me... Could this be a starting point for appeal... comments are welcome!

    ReplyDelete
  74. Got my results today and B is the only paper I (unsurprisingly) failed. I am surprised that they have awarded full marks for D1-1 for everyone, including people who were not affected by the technical issues (FR and DE papers), while they did not award any extra marks for this highly unfair paper B. Kudos to everyone that passed B outright this year

    ReplyDelete
    Replies
    1. B was equally unfair to all candidates. (At least largely... the markup of changes in the proposed claims was not the same in all languages.)

      D1-1 was not equal for all candidates. EN and FR candidates affected with different severity (some got 0 minutes extra, some not), DE candidates getting 30 minutes for free without any need. Stress due to false start for EN and FR candidates, not for DE candidates. Assume that need for equal treatment could only be met by neutralizing that part in full.

      RE:B!

      Delete
  75. So, results are out, and B (unsurprisingly) is the only paper I failed. I am surprised that they have awarded full marks for D1-1 to everyone, including people who were not affected by the technical issues (FR and DE papers), while they have not awarded any extra marks for the highly unfair paper B. anyway, kudos to everyone who passed paper B outright this year

    ReplyDelete
  76. In view of the examiners´report, do there seem to be easons and arguments possible for an appeal?

    ReplyDelete
  77. I also failed paper B and I am not surprised at all; it really was a mess. Now, studying the examiner's report, I cannot believe this is the expected solution to paper B. It is very unsatisfying in my opinion. If I were an examiner, I would say I considered the arguments, but not all of them were found to be convincing. Some points I want to emphasize and which could be useful for those of you who plan to appeal:

    1) Restricting claim 1 to a non-transparent lid is not what you immmediately come up with if you respect the "challenge" of the examiner's report to seek broadest possible protection. Any transparent or even partially transparent lid, or downwardly opening lid would circumvent the claim. If the EQE papers are considered to be a test whether candidates are fit for practice, I am not so sure that proposed claim 1 proves such fitness.

    2) I thought amending the claims to meet all the requirements of the EPC (cf Rule 24(3) IPREE) also includes unity of the invention. D2 discloses all the features of claim 1 as proposed, except for the opaque lid. Where is this, or a corresponding feature, found in claims 5 and 6? But maybe I am wrong and unity of the invention is not a requirement of the EPC that has to be met in the B paper after all. What do you think?

    3) D2 should be awarded points as CPA for caim 1. It is also a promising starting point as it has many features in common and also is directed to the same purpose of refuse containers. It is worth mentioning that proposed claim 1 is not limited to composting containers in particular and there seems to be confusion about a device and its possible uses in the examiner's report. The consequence of having D2 as CPA is that claim 1 would not be inventive, because the skilled person would use an opaque lid to improve the earthworm conditions if he was asked to adapt the container of D2 to an earthworm composting container (that the worms are light sensitive is CGL). As previously published on delta's case law, decision T 1294/16 confirms once more that for the purpose of Art 56 EPC, any document can be selected as CPA. The EQE testing the candidates fitness to practice should honour the current jurisprudence.

    4) I also think that the client's wish to not only limit claim 1 to drain holes can and should be respected by claiming "drain holes and/or water-absorbent material". Although all embodiments have drain holes, the information that water-absorbent material can be used to remove excess moisture is also given in the application and an embodiment with water-absorbing article is also described in the application. For me, the drain holes are not indispensable.

    5) Amending claim 6 to be a CII is not, in my opinion, enough to obtain a claim that is inventive over the information presented in par. 1, 5 and the table of D3. It is also questionable whether the inventive concept of optimizing composting conditions is realized in claim 6 since it encompasses, inter alia, organic refuse in the open space (light exposure) and refuse containers where no draining of moisture is possible. Claim 5, in contrast, expressly incorporates the mositure addition and removal, but also is silent on the dark conditions that should prevail in the container.

    ReplyDelete
    Replies
    1. My thoughts:

      1) I fully agree that the scope of protection of claim 1 is extremely limited, however: be careful that you do not start inventing things beyond the scope of the exam (partially transparent, downwardly opening)

      2) the non-transparent lid is not the invention, spraying water is.

      5) My biggest concern with claim 5 is that it is not specified which of the steps is actually computer implemented. GL F-IV 3.9: "Claims directed to CII should define all the features which are essential for the technical effect of the process which the computer program is intended to carry out when it is run (see F‑IV, 4.5.2, last sentence)."

      Delete
    2. Why would it be needed to specify which of the steps is actually computer implemented? It is a computer-implemented method, where the method has steps, so it is all fine: computer-implemented and each of the steps are all essential features, so OK for Art.84

      Delete
    3. @BS: I doubt unity of invention between claim 1 and claim 6 as it is in the Examiners' Report. Claim 6 needs the container of claim 1 for unity.

      NU1-6

      Delete
    4. @ AnonymousJune 23, 2021 11:03 am

      How else can we determine which of the steps can contribute to the inventive step of the claim, under the COMVIK-approach? Merely doing "something" in a computer cannot be inventive as such.

      https://www.epo.org/law-practice/legal-texts/html/caselaw/2019/e/clr_i_d_9_1_3_b.htm

      Delete
    5. The claim specifies that the method is CII-ed, and the claim specifies the steps of the method, so it is not "merely specifying something in a computer": it specifies the steps of the method!

      Delete
    6. @ AnonymousJune 23, 2021 11:24 am

      Interesting, I read the claim with the part of description "[024] One, more or all steps of the method can be implemented by a computer" in mind - which I should not have done.

      So, wouldn't it be an unnecessary limitation to claim that all steps of the method are computer-implemented?

      Delete
    7. Good point in view of your citation of [024]: that would allow:

      "An at least in part computer-implemented method ..."

      Delete
    8. @ Jan
      Regarding unity, I understand that the link between the independent claims is the spraying of water onto the refuse, but this is not enough. Checking the GL, F-V.3(ii), you will read the following:
      "Comparison of the common matter with the prior art at hand
      If common matter, namely subject-matter involving the same or corresponding technical features, is identified in the claims, it must be compared with the prior art at hand. If the common matter defines a non-obvious contribution over that prior art, it will involve "special technical features", and the inventions concerned will be so linked as to form a single general inventive concept. Otherwise, if the common matter is known or obvious from the prior art at hand, then the application lacks unity. "

      Apply this to D2 as the prior art at hand and you will see that the water spraying device a posteriori destroys the common matter in the claims. Then only the non-transparent lid is a special technical feature of claim 1, which is missing in the other dependent claims.

      Delete
    9. I don't remember any of the prior art documents directly disclosing "spraying of water onto the refuse". The cleaning device for spraying water is not disclosed as being used for spraying refuse to increase moisture content - which would be the inventive concept of the application of the client.

      Delete
    10. for = suitable for if in a device claim...

      your argument would hold for a use-claim - G 2/88

      Delete
    11. @ Roel van WoudenbergJune 28, 2021 10:45 am

      Thank you Roel for your input. Since the water spraying device of claim 1 is also suitable for adding moisture on the refuse, I am interetsed to hear if you then would also agree that the independent claims of the Examiner's report are lacking unity of invention a posteriori, in view of the container disclosed in D2, and that, as a result thereof, the proposed claims set does not comply with all the requirements set out by the EPC?

      Delete
    12. @BS: I share your doubt, but may for different reasons. Could you please give your complete non-unity argumentation, using the 3 independent claims (1 - container; 5 - method; 6 - CII method). What is the common matter, does that common matter make a contribution / does it have "special technical features"? (Guidelines F-V; note that the non-unity chapter has been significantly rewritten in the 2021 edition)

      Delete
    13. @ Roel van WoudenbergJune 28, 2021 3:44 pm
      Here is my complete reasoning for the lack of unity of invention (split over two comments):

      Claim 5 includes the step of adjusting the moisture of the refuse in a container. This guarantees that moisture is kept in a range where the earthworm are composting the refuse and also survive on hot and dry days.

      Claim 6 includes the step of recommending that the amount of moisture added to the refuse is changed if the earthworm population is not equal to a target value. Claim 6 does not say in what direction or by how much precisely the added amount should be changed, it does not say whether the recommendation is for earthworm populations larger or smaller then the target value, and the claim does not require the refuse to be in a container.

      From the description one does not learn anything about the proliferation rates of an earthworm population, only the decline in a refuse too dry to survive is disclosed. It thus appears that the recommendation of claim 6 is given for dry organic refuses for which the amount of moisture that is normally added is insufficient and needs adjustment. In that sense, an operator is prompted to increase the amount of moisture added to the refuse at time point tp to ensure the earthworm survival.

      Claim 5 and 6 therefore both address the particular problem of ensuring earthworm survival in hot and dry organic refuses to be processed. This is the common matter between claims 5 and 6.

      Claim 1 relates to a container for organic refuse, which is adapted to adjust the moisture in the refuse by draining excess water and by spraying additional water on the refuse. The container of claim is thus a suitable apparatus for carrying out the fertilizer production method of claim 5. They share the common matter of regulating the moisture content in the refuse in the container.

      A priori unity of invention appears to be given through the common effect of adding, or enabling the addition of, moisture to the organic refuse, thereby solving the common problem of ensuring earthworm survival in hot and dry organic refuses to be processed. A more general problem of providing a composting container for producing fertilizer with worms is not justified in view of the available prior art (par. 3) and the fact that neither claim 5 nor claim 6 refer to the use of the specifically claimed container (i.e. the container is not common matter).

      Delete
    14. A lack of unity of invention a posterior is established for the following reasons.

      Prior art D2 discloses a container with compartment suitable for holding organic refuse. The mesh floor is considered as a support with drain holes and the spray cleaning device is suitable for spraying water on the refuse, thereby adjusting the moisture.

      A container as disclosed in D2 thus obviously enables adding moisture to the refuse and implicitly solves the common problem to all independent claims stated above. The only distinguishing, special technical feature of claim 1 that can make an inventive contribution over the prior art D2 is the non-transparent lid. The associated technical effect is that of preventing light from entering the container and the technical problem is that of creating a more favourable (dark) environment for the development of earthworms in the refuse (par. 13).

      Although the container of D2 is suitable for the method of claim 5, D2 does not disclose the use of the container for producing fertilizer by composting the refuse with earthworms. No specific container is claimed in claim 5, step a), which therefore lacks a feature corresponding to the opaque lid.

      Steps b), second half, and step c) of claim 5 now are the only special technical features over the common matter. The combined effect is that excess water is drained and the refuse does not repelling the worms, which then continue effectively composting the refuse (par. 4). The associated technical problem is that of avoiding excessive moisture levels in the refuse for which the earthworms would stop assisting in the composting process of the refuse. This problem of ensuring attractive conditions for the compositing by the worms is not related with the problem of improving the development conditions for the earthworms. In so far as the description is concerned, there is no explanation about or hint to any possible relationship between the two problems. This confirms the lack of unity between claim 1 and 5 a posteriori.

      Claim 6 is not limited to the specifically claimed container or the spraying of water. The special technical features of claim 6 consist of the comparison of earthworm population data with a predetermined target population as a decision criterion to recommend adjustment of the added amount of moisture. The associated problem is to inform the operator whether the amount of added moisture should be changed at a specific point in time, in a way which requires minimum operator skill and involvement (par. 6-9). But the modalities of informing the operator about an added moisture adjustment are not related to the two above-mentioned problems and there are no corresponding features in claims 1 and 5 that would combine with the special technical feature of claim 6 (e.g. display, alarm, IR camera).

      Delete
    15. A similar analysis could start from D3 as the prior art at hand. D3 also discloses the composting (= processing) of organic refuse with earthworms and discloses adding moisture to the refuse, i.e. the water reservoir in the lower compartment which ensures a constant moisture level (this implies that in hot and dry conditions water from the reservoir is transferred to the refuse). Hence, also D3 anticipates the common matter and causes non-unity.

      Likewise, the Art. 94(3) Communication indicates that the spraying of water on the refuse would be obvious in light of the composting method in D1, which again would establish non-unity between the claims a posteriori (at least between claims 1 and 5 on the one hand and claim 6 on the other hand).

      Besides, claims 5 and 6 are formulated as two independent method claims. It is therefore to be verified whether claims 5 and 6 fall under one of the exceptions R 43(2). It appears that only exception iii) could apply, i.e. alternative solutions for a particular problem which cannot properly claimed in a single claim.

      However, the solutions that are proposed by independent claims 5 and 6 are not different or mutually exclusive, because both teach the addition of moisture. Moreover, the computer-implemented steps of claim 6 could be incorporated into claim 5 to specify the circumstances under which moisture is added. Therefore, R43(2) also seems to be violated.

      Delete
  78. Please excuse you, my english not so good. I wondering if DeltaPatents use new method for B course? Many peoples said me that Paper B now different than in last years. I not finding mocking papers, so I need course for that. Any you help me, please?

    ReplyDelete
  79. Hi. I am asking this question also on this blog, maybe someone can please help me with a jurisprudence or with an official marking rule of EPO. In the B paper one gets punished with 4 marks by adding the houseflies in claim 1 and loses all marks when doing the same mistake in claim 5. Wasn't there a marking practice not to double penalise candidates for the same mistake? Does it make really sense (=is allowable?) to have two different "punishments" for the same mistake in the same exam ? I would say this is very odd.

    ReplyDelete
    Replies
    1. Hi John,
      If I understand the appeal case law correctly, the double-penalization is only prohibited in cases where the questions or parts of the paper are interdependent. Since in the present case there are two independent claims, marks can be deduced separately for each claim. In your case, I would try to recover the 4 points in claim 1 by arguing that the claim to a container is not comprising the refuse itself. Thus, the refuse itself or populated with earthworms, eggs, larvae, or anything similar, is not part of the subject matter of the claim (i.e. does not define the matter = container for which protection is sought). Hence, there the eggs cannot be added subject matter for the container claim and Art. 123(2) EPC is not applicable. The only extent to which the refuse indirectly determines the properties of the container is via the holding support, but changing worms for eggs in the refuse does not affect the properties of the support, given in particular that housefly eggs are known additive for the refuse. Things are different for the method claim, where composting by earthworms or housefly eggs define two different methods.

      Delete
    2. @Bs: your argument "Thus, the refuse itself or populated with earthworms, eggs, larvae, or anything similar, is not part of the subject matter of the claim (i.e. does not define the matter = container for which protection is sought). Hence, there the eggs cannot be added subject matter for the
      container claim and Art. 123(2) EPC is not applicable" is not correct.
      Art.123(2) is NOT limited to the subject-matter of the claim.
      Art.123(2) relates to the application as a whole.
      Adding "optimally a dragonfly" somewhere in the description, claims, or drawings would also violate 123(2)!

      Delete
    3. Hi BS.

      is there any case law relevant for this case?
      I would like to study it a little bit before filing the appeal .

      Thanks !!

      Delete
  80. CII claim 6 of the Examiners' Report is not unitary with the device and method of amended claims 1 and 4 of the Examiners' Report.
    The Examiners' Report is silent about unity/non-unity.
    I have spend quite some time on considering it and added the container of claim 1 to the CII claim.

    NU1-6

    ReplyDelete
  81. That is of course true, but what I meant is that the change in the application has been made in the container claim and only in the context of the holding support. Third party interests are not jeopardized because the scope of the claim is not changed - they can expect a support for holding the refuse, be it with earthworms or housefly eggs. The applicant also does not improve his position, because all parts of the container claim are still the same. The skilled person is also not given any new information, other than what has already been disclosed in the claims and the description, namely that the support holds the refuse with earthworms, from which he infers with his CGL that it also holds the refuse with housefly eggs. The appearance of the worms in claim 1 is anyway artificial, in practice one would just mention the refuse. I agree that the active method step of populating the refuse with housefly eggs alone in the context of actually composting the refuse would be added subject matter, but in the context of a holding support I think you can argue that there is no extension of subject matter. Undisclosed disclaimers also add negative features for which there is no basis in the description, but they can be allowable because they restrict the claim scope. Here, the claim scope is not changed (no embodiments removed or added), so I'd say one could give it a try, at least if passing the paper depends on it.

    ReplyDelete
  82. Hi BS and many many thanks for your advice, it seems very logic!

    And there is one more aspect, referring to method claim 5.
    In the past papers the marking scheme would differentiate between infringements of Art. 123(2) which can be fixed by deletion of the wrong amendment and infringements which can not be fixed (the Art. 123(3) trap). In this case one could delete "optionally housefly" from claim 5 resulting in a narrower acceptable claim which has only worms..

    ReplyDelete
    Replies
    1. Hi John,
      referring to your question on relevant case law above, I can tell you that I have no personal experience of such a case (taking the EQE I have gained only limted experience over the past few years), but a quick look into the case law book shows that there might be cases which are close to the exam situation. For example, T 0235/99 (pt.2 of the reasons) also deals with the situation where a feature relating to the possible use of an apparatus has been amended (in this case by omission), but was not found open for objection under Art. 123(2) and (3) because the omission of this feature did not " extend the scope of the claim to include embodiments of the claimed apparatus which were previously excluded". This applies by analogy to the container of claim 1, for which it is irrelvant whether it is used for composting with worms or with eggs, or to simply hold the organic refuse.
      See also T 0835/97 (feature f) and T 0692/97 (pt. 2.1 of the reasons), also concerned with the omission of features aiming at an intended use of the device and not infringing Art. 123(2); the features are "of descriptive nature" rather than of "limitative nature". Possible that there is even more case law to find on that topic (e.g. specifically on replaced parts of features, more in line with your case).
      Hope that helps you with your case !

      Delete
  83. Hi John,
    The "optionally housefly" has no effect on the scope of the claim (only on its validity): an optional feature does not enlarge nor narrow the claim scope.
    I agree that amendments which do not get you in the trap should be penalized less than amendments that do get you in the trap. But I do not know whether that has also been the practice in marking paper B.

    ReplyDelete
  84. Great, many many thanks ! The decisions seem very close to my case. the only issue is with the fact that in the decisions "use features" are deleted from the claims whereas here the housefly eggs are added. I intend to say that the skilled person would derive unambiguously that housefly eggs could have been added to the refuse, this being already common knowledge disclosed by the application (the passage relating to background art).

    ReplyDelete
    Replies
    1. Yes, the use features are very often purpose-related and the purpose of the holding support in the container of claim 1 is to hold the organic refuse. If it is capable of holding the refuse with earthworms (which move), it is obvious that it is also capable of holding the refuse with eggs (which do not move). Whether earthworms are eggs are added to the refuse is irrelevant to the scope of claim 1, because it only relates to the intended "use" of the container, e.g. composting with earthworms or by addition of eggs.

      Delete
  85. By the way, does anybody have an idea why different solutions to claim 1 were marked so differently, e.g. the non-transparent lid full marks, the moisture detector -2 marks and the water container -8 marks? Isn't that very arbitrary, given the fact that in the ligth of the examiner's report all solutions are equally justified in the sense that they achieve the same goal of overcoming the objections? That is because:
    1) all solutions limit claim 1 to be novel over D2
    2) all solutions are respecting the client wishes
    3) all solutions are leading to a narrowed scope of protection and are easily circumventable
    4) all solutions have in common that the added distinguishing feature is not relied on in the IS attack against D1 as CPA (D2 never being seriously considered as a good alternative choice for the CPA, and even if D2 was selected the CPA, all solutins add features for which there wouldn't be a problem that is obvious to solve, e.g. dark conditions in the container which is more favorable for the survival of the worms in the manure (D2 does not mention why it would be a problem to replace the transparent lid by one that is not), measuring the removal of moisture through the tunneling mre accurately, or provide an alternative to the water tap connection).

    ReplyDelete
    Replies
    1. No clue...

      Also no clue why the most far-fetched solution scores best

      Delete
    2. I am not shure if the marking scheme in the examiner's report was really applied to all exams. My claim 1 looks totally different to the "model solution", i.e. I deleted the draining holes and included the moisture detector. I also performed (in my view) a rudimentary problem solution approach for IS of claims 1, 4 and 6, because I had only 15 minutes left. I nearly scored the total of 26 marks for IS arguments and ended with about 75 point in part B.

      --> no clue how marking was done but in my view there had been a reasonable compensation.

      Delete
    3. Page 8, chapter 3 of the Examiners' Report (marking of the amendments to the claims):

      "It is noted that full marks could be awarded for amendments that differ from those of the expected claim set. This is considered on a case-by-case basis."

      Page 10-11, chapter 5.1 (marking of the reply letter):
      "For an answer paper having a different set of claims, the letter of reply may differ, and the answer paper is considered accordingly."

      Delete
  86. Chapter 5.2, last sentence of the Examiners´Report states: "No marks are available for a claim that contains added matter."
    Isn´t some kind of double and tripple punishment?

    ReplyDelete
    Replies
    1. Yes , you did get punished very severely, you lost around 30 points or maybe more if you added "or housefly eggs" in claims 1 and 5. you didn't get any points for chapter "amendments of claims" and you didn't get any points for claim 5. You also lost 4 points in claim 1 . That's kind of new in a B Paper , isn't it ?

      Delete
  87. Does anyone know where to find model answers? The ones provided by EQE committee aren't so helpful and the candidate's answer doesn't look particularly good and no marks for this answer are given.

    ReplyDelete
  88. https://www.epo.org/law-practice/case-law-appeals/recent/d210008du1.html :

    7. Offensichtlich fehlerhafter Punkteabzug bei der Klarheitsbearbeitung

    7.1 Die Kammer kann nicht nachvollziehen, warum die Prüfungsarbeit der Beschwerdeführerin für die Bearbeitung der Frage der Klarheit keinen bzw. lediglich 1 Punkt von insgesamt 3 erreichbaren Punkten erhalten hat. Die Lösung der Beschwerdeführerin (siehe oben Ziffer IV(d)(bb)) enthält alle laut Prüferbericht notwendigen Angaben, wobei die Kammer es für praxisgemäß und damit unschädlich hält, dass auf Absatz [006] der Anmeldung lediglich Bezug genommen und dessen Inhalt nicht wortwörtlich wiedergegeben wurde. In diesem Zusammenhang ist außerdem anzumerken, dass die im Internet veröffentlichten exemplarischen deutschen und englischen Prüfungsarbeiten von Bewerbern - von denen grundsätzlich ausgegangen werden kann, dass sie eine akzeptable und im Wesentlichen vollständige Lösung enthalten ? vergleichbar sind (vgl. die deutsche Bearbeitung: "Der in [04] Ihrer Mitteilung vorgebrachte Klarheitseinwand gegen den früheren Anspruch 4, neu 5, ist durch die Ersetzung von 'Würmer' durch 'Regenwürmer' im geänderten Anspruch 5 behoben, siehe oben." zusammen mit der in Bezug genommenen Bearbeitung weiter oben in der Lösung "Dabei wurde der Begriff 'Würmer (8)' durch 'Regenwürmer (8)' ersetzt. Der Gebrauch der Begriffe ist durch [06] klar offenbart 'Im Zusammenhang mit der vorliegenden Erfindung werden die Begriffe Würmer und Regenwürmer austauschbar gebraucht und bezeichnen Regenwürmer' und die Änderung damit zulässig. Die Änderung behebt weiterhin den Klarheitseinwand in Ihrer Mitteilung." sowie die englische Bearbeitung "The examiner alleged that previous claim 4 lacked support in the description as it related to 'worms' in general. As claim 4 is amended to state that the 'worms' are earthworms, this objection may be set aside"). Ferner ist der Kammer aus einer anderen Beschwerde die Lösung des Bearbeiters Nr. 69735 bekannt ("The feature 'worms' has been replaced by 'earthworms' based on description par. 6, in this way overcoming also the objection of clarity directed to 'worms' ? 'earthworms'"), welche mit derjenigen der Beschwerdeführerin ebenfalls vergleichbar ist und die volle Punktzahl erhalten hat.

    7.2 Somit scheint den Prüfern bei der Bewertung der Klarheitsbearbeitung offensichtlich ein Fehler unterlaufen zu sein; die Vergabe von keinem bzw. nur einem Punkt ist für die Kammer unter keinem Gesichtspunkt nachvollziehbar. Ob freilich die volle Punktzahl für die Bearbeitung der Beschwerdeführerin vergeben werden muss, wie von ihr gefordert, oder nicht, ist von der Prüfungskommission und nicht von der Beschwerdekammer zu entscheiden; letztere kann sich grundsätzlich nicht an die Stelle der ersteren setzen. Die Angelegenheit ist daher insofern zur erneuten Beurteilung im Sinne von Artikel 6 (5) VEP an die Prüfungskommission zurückzuverweisen.

    [...]

    ReplyDelete
    Replies
    1. 10. Verletzung des Gleichbehandlungsgebots aufgrund Nichtberücksichtigung einer fehlenden Markierung einer Anspruchsänderung

      10.1 Die Beschwerdeführerin hat schlüssig geltend gemacht, dass in der deutschen Aufgabenstellung in Anspruch 5 das Merkmal "durch Besprühen des Abfalls mit Wasser" fälschlicherweise nicht als durch die Mandantin vorgenommene Änderung fett markiert wurde und der Beschwerdeführerin für die Identifizierung dieses Fehlers ein deutlicher Zeitverlust entstanden ist, welcher zudem durch die fehlende Möglichkeit in der Prüfungssoftware, die Ansprüche auszudrucken, nebeneinander zu legen und zu vergleichen, verstärkt wurde. Ein Prüfling, der mit der französischen Angabe arbeitete, in welcher der beschriebene Fehler nicht enthalten war, hatte also im Vergleich zu einem Prüfling, der mit der deutschen Angabe arbeitete, einen nicht unerheblichen Zeitvorteil. In der Nichtberücksichtigung dieses Umstands zugunsten der Beschwerdeführerin sah diese einen Verstoß gegen das Gebot der Gleichbehandlung (vgl. oben Ziffer IV(f)).

      10.2 Die Beschwerdekammer in Disziplinarangelegenheiten überprüft gemäß ihrer ständigen Praxis die Einhaltung des Gebots der Gleichbehandlung von Bewerbern der Europäischen Eignungsprüfung als höherrangiges Recht (vgl. D 2/95; D 14/95; D 10/97; D 5/99; D 19/04). Dieses Gebot verlangt, dass bei steter Orientierung am Gerechtigkeitsgedanken Gleiches gleich und Ungleiches seiner Eigenart entsprechend verschieden zu behandeln ist (vgl. D 19/04, Gründe 5.1). Vergleichbare Sachverhalte dürfen demnach nicht unterschiedlich behandelt werden, es sei denn, dass die Differenzierung objektiv gerechtfertigt, d. h. in einer gegebenen Situation nach Art und Ausmaß sachlich begründet ist. Ein zu beanstandender Rechtsfehler liegt dann vor, wenn die Prüfungsbedingungen so gewählt sind, dass sie einen Teil der Bewerber benachteiligen, ohne dass hierfür ein sachlicher Grund erkennbar wäre (vgl. D 3/95, Gründe 3; D 10/97, Gründe 4.1; D 11/19, Gründe 8.3.3 a)).

      10.3 In den in Ziffer 10.1 oben beschriebenen Umständen für die Bearbeiter der französischen Aufgabenstellung einerseits und für die Bearbeiter der deutschen Aufgabenstellung (und im Übrigen auch der englischen Aufgabenstellung, in der der gleiche Fehler enthalten ist, wie die Kammer festgestellt hat) andererseits ist eine Ungleichbehandlung festzustellen, ohne dass hierfür eine objektive Rechtfertigung erkennbar ist. Weder aus dem Prüferbericht noch aus dem Schreiben des Prüfungssekretariats vom 21. Juni 2021 ist außerdem zu erkennen, dass diese unterschiedlichen Umstände bei der Bewertung der Arbeiten der Bewerber der deutschen Fassung, wie die Beschwerdeführerin, berücksichtigt wurden.

      10.4 Die Bewertung, wie der der Beschwerdeführerin durch die Ungleichbehandlung entstandene Nachteil zu berücksichtigen ist, zum Beispiel durch Vergabe von zusätzlichen Punkten, obliegt prinzipiell nicht der Kammer, sondern der Prüfungskommission. In der Konsequenz ist es also der Prüfungskommission überlassen, nach Zurückverweisung der Angelegenheit einen nach Art und Ausmaß in der gegebenen Situation angemessenen Ausgleich zu finden und etwa die Benotung gemäß Artikel 6 (5) Satz 2 VEP entsprechend zu korrigieren. Die Beschwerdeführerin kann dann gegebenenfalls im Rahmen einer erneuten Beschwerde die von der Prüfungskommission getroffenen Anordnungen überprüfen lassen (vgl. insofern sowie auch zu möglichen Gesichtspunkten, die bei der Findung eines angemessenen Ausgleichs insbesondere für einen Zeitverlust Berücksichtigung finden können, D 11/19, Gründe 8.3.5).

      [...]

      Delete

    2. 12.1 Aus dem oben Ausgeführten ergibt sich, dass dem Hauptantrag der Beschwerdeführerin, die Prüfungsaufgabe B mit mindestens der Note "NICHT BESTANDEN MIT AUSGLEICHSMÖGLICHKEIT" zu bewerten, nicht entsprochen werden kann. Dagegen ist aus den in Ziffern 7 und 10 ausgeführten Gründen der Hilfsantrag begründet und entsprechend die angefochtene Entscheidung aufzuheben und die Angelegenheit an die Prüfungskommission zur erneuten Entscheidung unter Berücksichtigung der rechtlichen Beurteilung der Kammer (vgl. Artikel 24 (4) Satz 1 VEP in Verbindung mit Artikel 22 (3) VDV und Artikel 111 (2) Satz 1 EPÜ) zurückzuverweisen.

      [...]

      Entscheidungsformel

      Aus diesen Gründen wird entschieden:

      1. Die angefochtene Entscheidung der Prüfungskommission wird aufgehoben.

      2. Die Angelegenheit wird zur erneuten Entscheidung an die Prüfungskommission zurückverwiesen.

      3. Die Rückzahlung der Beschwerdegebühr wird angeordnet.

      Delete

  89. Hello Roel.

    the same mistake was also in the EN version: no bold.
    My appeal is with the DBA now but I didn't raise this issue yet.
    Do you know whether the DBA accepts that new reasons are filed also later (like now)?

    And: do you think the same principle could also apply to the translation of the word "non-transparent" (EN) versus "opaque" (FR). I sat the exam in English and the wording "non-transparent" was Greek to me (Translucent is non transparent but lets light penetrate..., and in fact "non-transparent" is a disclaimer, the word as such does not exist in any English dictionary).

    The French "opaque" doesn't let any doubts open , it means clearly that no light can go through.

    And a last question: was it indeed like in the published marking scheme that there were no points at all if a candidate had added "or housefly eggs" in claims 1 and 5?

    Thank you and have a nice evening.



    ReplyDelete
    Replies
    1. I do not know until when the DBA will take fresh submissions into account. If RPBA2020 apply, it may be difficult to get things in late and especially only during oral proceedings... But in any case, the later the less chance, so I would suggest that you submit it a.s.a.p., and give reasons for the late submission. Good luck.

      By the way, note the difference between a serious error and an error of judgement / difference in interpretation / discretion w.r.t. the marking scheme - see the consistent case law of the DBA on the scope/extent of their competence (of which D 2/21 gives a nice overview - see my post in the Pre-Exam blog).

      Delete
    2. D 37/21 - https://www.epo.org/law-practice/case-law-appeals/recent/d210037eu1.html

      Facts VI: late submissions:

      VI. With letter dated 4 October 2021 the appellant referred to the above letter of the Examination Secretariat and gave further arguments why the Examination Board made a serious and obvious mistake in the marking of her answer paper.

      reason 8:

      8. Beside her appeal, the appellant also relies on her further submissions, in particular the information available from the letter of the Secretariat dated 10 August 2021 mentioned in point V. above. The Board sees no reason for not taking these submissions into account (Article 114(2) EPC in conjunction with Article 25(1) RDR). The statement of the Examination Board is relevant for the case and the appellant could not have submitted it with the appeal.

      Delete
  90. Several appeals have been successful (most resulting at least in a re-marking with some instructions from the DBA):

    D 0012/21 () of 16.2.2022 - https://www.epo.org/law-practice/case-law-appeals/recent/d210012eu1.html:
    3. In a notice of 10 March 2021, the Examination Secreteriat informed candidates that it had received a number of emails from candidates describing their circumstances and individual experience in relation to the pre-examination or the papers of the main examination, and that that their emails would be forwarded to the Examination Board. However, the Examination Board's letter dated 20 July 2021, informing the appellant of the decision that the appellant had not been successful in the European qualifying examination, only contains the results of the marks awarded by the Examination Board to the papers sat by the appellant. There is no mention of the appellant's complaint, neither that it was actually taken into consideration nor what the result of any such consideration was. Similarly, the marking sheets of the examination committees, enclosed with the said letter only show the number of marks allocated by the two markers respectively. Again, no indication whatsoever can be derived from the schedule of marks that the appellant's complaint was considered in the marking procedure. Since the appealed decision did not show that the appellant's complaint was taken into consideration, Rule 19(4) IPREE was not complied with.
    4. In the absence of a reasoned decision pursuant to Rule 19(4) IPREE, it is not for the Board to evaluate the merits of the appellant's complaint, but it will be for the Examination Board to decide - again - on the appellant's paper B considering the appellant's complaint. Therefore the decision under appeal has to be set aside and the case remitted to the Examination Board for a new decision.

    D 0022/21 () of 17.2.2022 - https://www.epo.org/law-practice/case-law-appeals/recent/d210022eu1.html:
    16. The re-evaluation should be based on a marking scheme which is fairly proportional in the sense that it leaves some room for the discretional awarding of marks where it appears equitable for recognising expected solutions, in particular where such expected solutions cannot be awarded the full marks for some reason. The marking scheme should either avoid multiple deductions for the same error or should only apply it in properly justified circumstances. Multiple deduction of marks does not appear justified where it cannot be reasonably expected that the candidate recognises the error when repeating it, for example where the same amendment is introduced in the claims, or where the same arguments are provided for the same amendment made in different claims. Alternatively, the marking scheme should provide the explanation why multiple deductions of marks for the same error may be justified in view of the facts of the paper. The Board does not exclude that also the same amendment may be given a different weight, depending on the totality of the paper, in particular such details as the type of the claims amended, the effect of an amendment on other features and the disclosure serving as the possible support for the amendment in question. The Board again refers to decision D 0009/11, point 7.2.1 of the Reasons and emphasizes that separate deductions for claims and corresponding arguments is not necessarily a double penalty and as such not necessarily unfair. The marking is only unfair to the extent that equal errors are penalised unequally, or repeated errors are penalised repeatedly. Depending on the circumstances of the error, it may possibly result in a further - and possibly different - deduction of marks.

    ReplyDelete
    Replies
    1. And:

      D 0019/21 () of 21.2.2022 - https://www.epo.org/law-practice/case-law-appeals/recent/d210019eu1.html:
      24. The Board agrees that some proportionality of the penalties, i.e. the deductions can be expected in the marking scheme. That proportionality of the awarded marks is a recognised principle of the marking scheme is also apparent from the Examiner's Report, point 1.8, 2nd paragraph: "...the number of available marks corresponds to the difficulties of each challenge or the complexity of the expected amendment. In other words, more difficult challenges were awarded more marks than easier challenges.". This would seem to dictate some degree of proportionality also in the determination of the applicable deductions. The Board has doubts that the need for proportionality is properly observed in the present marking scheme, in view of the significant potential loss for the amendments, in particular the "housefly eggs" amendment, combined with the fact that the corresponding deductions could completely offset correct solution elements. On the other hand, the Board observes that even with a more proportional marking scheme the appellant may not necessarily achieve more marks for her claim amendments. As derivable from her tabular summary of the presumed deductions, these surpassed significantly the number of foreseen marks for the expected amendments. Under such circumstances correct amendments will not necessarily result in the awarding of marks. This is perfectly compatible with the expected proportionality of the marking scheme.
      Order
      2. The case is remitted to the Examination Board with the following order:
      the Examination Board is instructed
      a, to determine the applicable compensation for the answer paper of the appellant for the Paper B of the European qualifying examination 2021 in respect of the unequal treatment caused by the erroneous examination paper, and
      b, to instruct the competent Examination Committee to perform a re-marking of the appellant's answer paper for the Paper B, also taking into account the determined compensation, and
      c, to award a grade to the answer paper under Rule 6(3) IPREE accordingly.

      Delete
    2. And:

      D 0037/21 () of 21.2.2022 - https://www.epo.org/law-practice/case-law-appeals/recent/d210037eu1.html:
      7. In the present case, the appellant candidate set out plausibly that the closing of the lockdown browser window was unexpected and that the interruption of the examination in this manner cannot be imputed to her. The Board has no reason to doubt that she followed the instructions of the Examination Secretariat and took the expected steps in the given situation. The appeal itself does not quantify the effective time loss, but the Board finds it plausible that the time loss caused by the incident was not insignificant. The Board also accepts that even if the actually measurable time loss between the closing of the window and the subsequent successful logging in of the candidate may not have been excessively long, the additional stress effectively prevented her from concentrating on the paper to the same extent as before.
      25. Therefore, the Board decides to remit the case to the Examination Board with the order to determine the applicable compensation of the candidate in respect of the incident reported during the Paper B examination, and to undertake a new marking of the appellant's Paper B of the European qualifying examination 2021 under its powers pursuant to Article 6(5), last sentence, REE and to award a grade to the appellant on the basis of the re-marking.
      26. The re-evaluation should take into account the incident reported by the appellant, and the results of the marking should indicate clearly how this had been taken into account. To the extent possible, any compensation should be based on objectively determined facts and parameters. This does not exclude that less exact conditions are also taken into account, such as the stressfulness of the situation following the incident. For example, the 15 minutes mentioned in the letter of the Secretariat can also be reviewed, if it is established that it does not represent correctly the effective time loss, contrary to the hypothetical assumptions of the Board, as set out in point 23 above.

      Delete
    3. Today, 2 new decisions on B 2021 were published on the EPO website:

      D 0039/21 () of 28.2.2022 - https://www.epo.org/law-practice/case-law-appeals/recent/d210039eu1.html

      D 0029/21 () of 28.2.2022 - https://www.epo.org/law-practice/case-law-appeals/recent/d210029eu1.html

      Delete
    4. D 0039/21 () of 28.2.2022 - https://www.epo.org/law-practice/case-law-appeals/recent/d210039eu1.html :

      "Arbitrary features of the marking scheme and double penalty

      9. The Board must establish that the marking scheme contained features that appeared arbitrary and the Examiner's Report did not exclude the possibility of double penalty. The Board refers to decision D 0022/21, points 8 and 9 of the reasons, and explicitly concurs with the reasons given in support of these findings of the present Board."

      "Correct solution elements remain unrewarded

      10. The appellant submitted that his paper contained several elements of the expected amendments that were foreseen by the Examiner's Report, and which did not attract any marks because of the total loss caused by the "housefly eggs" penalties. Specifically, he removed the lower compartment in claim 1, and his claim 5 corresponded very much to the expected solution, apart from the "housefly eggs" amendment. The Board considers that such a result is almost an inevitable consequence of a marking scheme where deductions can be made for various reasons. Candidates cannot expect that they will be awarded the full marks for certain partial solutions under any circumstances, even if these are doubtless correct on their own. According to Rule 24 IPREE, in the Paper B part of the EQE candidates are expected to respond to all points raised in the official communication and to provide amended claims that meet the requirements of the EPC, i.e. all requirements of the EPC. Thus answering Paper B cannot be reduced to the simple exercise of collecting marks for certain solution elements that are derivable from the Examiner's Report. It is also required that the totality of the claims and the corresponding arguments constitute a complete and in itself consistent solution.

      11. Still, the Board accepts that the particularity of the marking scheme that all marks for a claim and amendment arguments may be lost by a single error may be perceived as unfair. The deductions may be perceived as disproportionate, given that the marking scheme may not sufficiently distinguish between candidates. For this reason, the Board has doubts that the need for proportionality is properly observed in the present marking scheme. In this respect the Board refers to decision D 0022/21, points 11 and 12 of the reasons, and explicitly concurs with the reasons given in support of the above findings of the present Board.

      12. In view of these details of the marking the Board cannot come to the conclusion that the marking of the "housefly eggs" amendment can be considered as fair in all respects. On the other hand, the Board also cannot establish beyond any doubt that the low number of marks awarded to the appellant indeed resulted from the unfair marking of the "housefly eggs" amendment, as argued by the appellant. As it is well known and also apparent from point 3 of the Examiner's Report for the Paper B 2021, deductions could be attributed to a number of factors, and not only for lack of support for the purposes of Article 123(2) EPC. Thus retroactively identifying the deductions effected in the appellant's answer paper specifically for the "housefly eggs" amendment would only be possible, if at all, if the Board scrutinized the totality of the Paper B (including the Examiner's Report) and the answer paper of the appellant, and effectively performed a complete marking of his answer paper. As explained above in point 4, such an exercise is beyond the competence of the DBA."

      Delete
    5. D 0039/21 () of 28.2.2022 [continued]

      "Request for re-marking and award of grade

      14. The appellant further requests that his answer paper be awarded a PASS or at least a COMPENSABLE FAIL grade. The Board follows the case law of the DBA (see e.g. decisions D 0024/17, point 15 of the Reasons, D 0013/17 and D 0016/17, point 4 of the Reasons) and considers that the Board itself cannot perform the re-marking requested. An assessment of the appellant's answer paper for determining the marks to be awarded would be equivalent to a review of the marking on the merits and thus would require value judgments which, according to the established jurisprudence (following D 1/92, OJ EPO 1993, 357), falls outside the competence of the Board, as already stated above in points 4 and 12. This is not changed by the remaining very short time until the next Paper B examination. Therefore, the Board decides to remit the case to the Examination Board with the order to instruct the competent Examination Committee to undertake a new marking of the appellant's Paper B of the European qualifying examination 2021 under its powers pursuant to Article 6(5), last sentence, REE and to award a grade to the appellant on the basis of the re-marking. However, this does not mean that the re-marking should necessarily achieve any given number of additional marks.

      15. The re-evaluation should be based on a marking scheme which is fairly proportional and avoids multiple penalties, according to the principles as set out in decision D 0022/21, point 16 of the Reasons. These principles are also supported by the present Board.""

      Delete
    6. While D 39/21 more-or-less reiterated arguments from the decisions cited above, D 29/21 did not only reiterate those arguments but also considered the differences between Wiseflow for Windows and for MAC to be an unequal treatment of candidates due to the differences of the editing functions.

      D 0029/21 () of 28.2.2022 - https://www.epo.org/law-practice/case-law-appeals/recent/d210029eu1.html

      "Unequal treatment of candidates due to the differences of the editing functions

      15. The appellant submitted that he had serious technical issues in the examination which he had duly reported. He referred to decision D 11/19, where the DBA concluded that different technical conditions during examination may amount to an unjustified unequal treatment of candidates. These problems of the appellant were the following:

      (a) [...]

      (b) The editor of the Wiseflow platform did provide for copy-pasting texts. However, when the text was pasted into the editor, numerous formatting errors would be present in the pasted text that made the text practically illegible. The Wiseflow version running on Microsoft Windows offered a copy-and-paste function that avoided these formatting errors (the key combination CTRL + SHIFT + V). On the other hand, this function was not available in the Wiseflow running on a MAC platform.

      (c) It was self-evident that the texts were far easier to read and errors were less likely to occur when reading the text without those formatting errors that occurred for MAC OS users, and the appellant had highlighted the issue to the Examination Secretariat. However, the issue still persisted during the Paper B. The appellant also indicated this on his answer paper and also reported in a separate e-mail to the Secretariat.

      (d) The problem of formatting errors could be partly eliminated by the formatting removing (Tx) function of Wiseflow. Still, this resulted in approximately one formatting error per line of text. In this manner reading the edited text was quite distracting and far more difficult to read, as compared with an error-free text. A simple calculation showed that reformatting and correcting errors must have taken a significant time. 16 minutes would be a good estimate, even if allowing for simplifications, e.g. only assuming 3 seconds to find and correct each formatting error. Calculating with 0.5 marks per minute, the appellant would be expected to have lost at least 8 marks, as compared to users of the Windows operating system."
      "

      Delete
    7. D 0029/21 () of 28.2.2022 [continued]

      "16. The Board finds the appellant's arguments credible that the differences between the Windows and MAC platforms caused significant differences in the perceived difficulty of the editing in Wiseflow. It is also clear that the online examination could not have been done without the editing of the texts of the paper. Furthermore, from the submissions of the appellant it appears that he was never told that sitting the Paper B would not be possible or recommended on a MAC based device. The Board considers that it cannot be expected from the Secretariat to provide tools that work on just any operation system. However, the Board considers it to be generally known that MAC OS based computers are a recognised and widespread class of computers, representing practically the only realistic option to Windows systems (at least for normal consumers). Thus it was reasonable for candidates to expect that Wiseflow will properly work also on MAC OS. By contrast, it would not have been reasonable to expect from candidates to learn how to use a Windows OS based device only for the purposes of the EQE examination. Put differently, candidates could have reasonably expected that the online examination can be absolved with the IT infrastructure they were familiar with.

      17. The Board is aware that it is practically not possible to provide perfectly equal conditions for all candidates under any circumstances. Some differences always remain and have to be accepted (see e.g. D 0011/19, point 8.3.3(c) of the Reasons, referring to different examination venues and the inevitable differences resulting therefrom). The Board considers that in view of the fact that the editing functions played a key role in answering the paper, the differences of the editing functions under the two operating systems, as explained by appellant, represent a difference that is not objectively justified. On this basis, the Board holds that the appellant was disadvantaged through the use of the MAC OS without good reason, and this disadvantage must be compensated.

      18. Similarly to the compensation of the unequal treatment caused by the erroneous examination paper, the appropriate compensation must be determined by the Examination Board. As suggested by the appellant, an estimate of the effective time loss may be considered as an objective measure of the disadvantage (see also D 0011/19, point 8.3.5 (c) of the Reasons, discussing estimated time loss as a possible measure of a disadvantage, even if inevitably imprecise). However, the Examination Board may consider other factors to be a more suitable measure for the disadvantage."

      "Order

      [...]

      2. The case is remitted to the Examination Board with the following order:

      a, the Examination Board is instructed to determine the applicable compensation for the answer paper of the appellant for the Paper B of the European qualifying examination 2021 in respect of the unequal treatment caused by (i) the erroneous examination paper and additionally the applicable compensation caused by (ii) the differences in the editing functions available to the candidates depending on the used operating system, and

      b, to instruct the competent Examination Committee to perform a re-marking of the appellant's answer paper for the Paper B, also taking into account the determined compensation, [...]"

      Delete
    8. Note that the DBA does NOT impose "PERFECTLY equal conditions":

      "17. The Board is aware that it is practically not possible to provide perfectly equal conditions for all candidates under any circumstances. Some differences always remain and have to be accepted (see e.g. D 0011/19, point 8.3.3(c) of the Reasons, referring to different examination venues and the inevitable differences resulting therefrom)."

      Delete
    9. And also in the following:

      D 0011/21 () of 28.2.2022 - https://www.epo.org/law-practice/case-law-appeals/recent/d210011eu1.html

      D 0027/21 () of 25.2.2022 - https://www.epo.org/law-practice/case-law-appeals/recent/d210027eu1.html

      it si reasoend that:

      5. [...] The Board observes that these latter objections were taken into account by the present marking scheme, given that the various parts of the paper could not be marked with less than 0 marks, irrespective of the total possible deductions applicable for the relevant part. These principles were further developed by decision D 0011/19, which held that separate deductions for erroneous claim amendments and additionally for the corresponding arguments by a candidate in support of such erroneous amendments do not necessarily mean an unfair double penalty. Under the circumstances such additional deductions may well be justified, given that the expected arguments give the candidate an opportunity to review its own assessment concerning the admissibility of an amendment, for example for the purposes of Article 123(2) EPC (D 0011/19, points 7.2.1 of the Reasons, in the context of Paper B).

      7. According to the Examiner's Report, accepting these suggested amendments of the client - in the following the "housefly eggs" amendment - was an error. If a candidate kept these amendments, various deductions resulted. The potential deductions caused by the amendment could affect the marks for the claims and the amendment arguments as well (points 3 and 5.2 of the Examiner's report). All claims, except Claim 1 could lose all marks, and all marks for amendment arguments could be also be lost.

      8. As a result, if a candidate erroneously assumed the admissibility of the housefly egg amendment - but otherwise kept the expected claim structure - , this could result in the loss of 26 marks for this error alone. Assuming that a candidate took up the housefly egg also in claim 6 could increase the potential loss up to 33 marks, and even up to 37 marks, looking apart from other potential deductions that could have been also caused by the insertion of the "housefly eggs" in the claims. For details of the calculation, reference is made to D 0022/21, point 6 of the Reasons. Assuming that the error only affected claims 1 and 5, i.e. assuming the seemingly most probable consequence if the error in the client's proposal was not recognised, the potential loss was 26 marks. The Board finds that the potential loss could be significant, if not excessive. In view of this significant potential loss, the marking must be fair.

      12. Still, the Board accepts that the particularity of the marking scheme that all marks for a claim and amendment arguments may be lost by a single error may be perceived as unfair. The deductions may be perceived as disproportionate, given that the marking scheme may not sufficiently distinguish between candidates. For this reason, the Board has doubts that the need for proportionality is properly observed in the present marking scheme. In this respect the Board refers to decision D 0022/21, points 11 and 12 of the reasons, and explicitly concurs with the reasons given in support of the above findings of the present Board.

      Delete
  91. Hi Roel,

    regarding D39/21, could you please guess why the DBA makes this statement:

    "The Board informed the appellant [...] .. other arguments as set out in points
    VI(a) ... were unlikely to succeed. These would appear to require a complete re-examination of the Paper B and as such would appear to be beyond the
    competence of the Disciplinary Board of Appeal."

    Below point VI(a) of the appellant:

    VI a):

    The Examiner’s Report was in error to penalise the
    “housefly eggs” amendment. The disclosure in the
    application of the Paper B permitted this amendment.
    Case law made it clear that the whole content of the
    application contributes to the disclosure. This was a
    serious and obvious mistake.

    I think the appellant was actually right and I don't see why is not the competence of the DBA to check whether the Paper B was well drafted.

    Clearly , this year , in order to check the "fit to practice" of the candidates the Examination Board over-stretched Art. 123(2).

    DBA should, in my opinion, give an opinion whether adding the "or housefly eggs" actually infringes Art. 123(2).

    The "or housefly eggs" doesn't have anything to do with the container.
    The "or housefly eggs" can only influence or determine to the properties of the organic refuse.

    If I say "container for organic refuse" OR "for LNG" then well, yes , I agree that the purpose may affect the subject matter claimed . An LNG container must obviously have different properties when compared to a container for organic refuse.
    These are definitely two different things .

    But why would a skilled person design a "container for organic refused POPULATED with earthworms" in a different way from a "container for organic refused POPULATED with housefly eggs" ??
    Isn't it the same container ?

    In my opinion it is a big mistake if DBA does not tackle this issue.

    I hope I do not offend anyone if I say that this B-Paper was actually not optimal although the Examination Board had two years time to double check it !

    I do not even mention the linguistic challenges like "non-transparent"/"undurchsichtig"... etc


    Cheers!


    ReplyDelete
    Replies
    1. I think the reason is that their competence is limited in view of REE Art.24(1):

      "(1) An appeal shall lie from decisions of the Examination Board and the Secretariat which adversely affect the appellant, but ONLY ON THE GROUNDS that this Regulation or any provision relating to its application has been infringed."

      See reason 4 and 5 of D 39/12 (and basically every EQE appeal decision from the DBA):

      "4. In accordance with Article 24(1) REE and the consistent case law of the Disciplinary Board of Appeal (hereinafter "the DBA"), which followed decision D 1/92 (OJ EPO 1993, 357), decisions of the Examination Board may in principle only be reviewed for the purposes of establishing that they do not infringe the REE, the provisions relating to its application, or higher-ranking law. It is not the function of the DBA to reconsider the entire examination procedure on the merits. This is because the Examination Committees and the Examination Board have some latitude in their evaluation which is subject to only limited judicial review by the DBA. Only if the appellant can show that the contested decision is based on serious and obvious mistakes can the DBA take this into account. The alleged mistake must be so obvious that it can be established without reopening the entire marking procedure. This is for instance the case if an examiner is found to have based his evaluation on a technically or legally incorrect premise upon which the contested decision rests (D 2/14). Another example of an obvious mistake would be a question whose wording is ambiguous or incomprehensible (D 13/02). All other claims to the effect that the papers have been marked incorrectly are not the responsibility of the DBA. Value judgments are not, in principle, subject to judicial review (see e.g. D 1/92, supra, points 3 to 5 of the Reasons)

      5. On the other hand, in exceptional cases the DBA recognised that unfair marking may constitute an obvious and serious mistake. Decisions D 0016/17 and D 0013/17 made it clear that double penalisation (Doppelbestrafung), namely where a false response may have an effect on other parts of the examination paper and thereby may lead to a further loss of marks for the same error, cannot be considered to fulfil the requirements for a fair marking as developed in the case law (point 3.7.1 of the Reasons in both decisions, concerning the marking of Paper A). These decisions also held that excessive, i.e. unreasonably high deductions for certain errors may also be considered to constitute unfair marking (Reasons 3.4, last paragraph in both decisions, in the context of potential deductions that could surpass the maximum possible marks). The Board observes that these latter objections were taken into account by the present marking scheme, given that the various parts of the paper could not be marked with less than 0 marks, irrespective of the total possible deductions applicable for the relevant part. These principles were further developed by decision D 0011/19, which held that separate deductions for erroneous claim amendments and additionally for the corresponding arguments by a candidate in support of such erroneous amendments do not necessarily mean an unfair double penalty. Under the circumstances such additional deductions may well be justified, given that the expected arguments give the candidate an opportunity to review its own assessment concerning the admissibility of an amendment, for example for the purposes of Article 123(2) EPC (D 0011/19, points 7.2.1 of the Reasons, in the context of Paper B)."

      Delete
    2. As to your:

      "But why would a skilled person design a "container for organic refused POPULATED with earthworms" in a different way from a "container for organic refused POPULATED with housefly eggs" ??
      Isn't it the same container ?"

      Art.123(2) is not about what the skilled person COULD all consider, but what is directly and unabiguously disclosed.
      So, even IF it would be the same container and considering it a suitable for, adding a reference in a (suitable) for part of the claim to non-disclosed subject-matter is an unallowable amendment.

      By the way, the Art. 123(2) argument is not different whether you consider the container to be "suitable for" or whether you consider the worms to be in the container, i.e., is the claim to be considered:
      "container (suitable) for organic refuse with earthworms in the organic refuse"
      or
      "container (suitable) for organic refuse, the container being populated with earthworms"

      Delete
    3. @John: but see reason 2 of D 0041/21 () of 1.3.2022, a successful appeal to paper C 2021.

      https://www.epo.org/law-practice/case-law-appeals/recent/d210041eu1.html

      "Request that the contested decision be set aside

      1. In accordance with Article 24(1) and (4) REE and the Disciplinary Board's of Appeal consistent case law (following D 1/92, OJ EPO 1993, 357), decisions of the Examination Board may in principle only be reviewed for the purposes of establishing that they do not infringe the REE, the provisions relating to its application, or higher-ranking law. It is not the function of the Disciplinary Board of Appeal to reconsider the entire examination procedure on the merits. This is because the Examination Committee and the Examination Board have some latitude in their evaluation which is subject to only limited judicial review by the appeal board.

      Only if the appellant can show that the contested decision is based on serious and obvious mistakes can the Board take this into account. The alleged mistake must be so obvious that it can be established without reopening the entire marking procedure. An example of an obvious mistake would be a question whose wording was ambiguous or incomprehensible (D 13/02). That would be clear straight away, without any reference to marks awarded, from the meaning that common sense would ascribe to the wording of the question concerned.

      2. Even if according to its case law it is not the function of the Disciplinary Board of Appeal to reconsider the entire examination procedure on the merits, what would be necessary in order to evaluate and conclude whether more or at least two more marks were to be awarded, the Board considers the appeal to be allowable. This conclusion can be arrived at on the basis of the appellant's submissions in the statement of grounds of appeal by a comparison between the marking scheme and the excerpt from the appellant's answer paper without reopening the entire marking procedure. The appellant demonstrated in the statement of grounds of appeal that the answer regarding Paper C, claims 1 and 3 complied with the Examiners' Report using the correct citation of the specific reference in the relevant documents. There are only a few minor deviations and differences compared to the Examiners' Report as set out below (points 3. and 4.) and also pointed to by the appellant itself, which, however, do not render the answer wrong or incomplete.

      [Detailed reasons follow in 3.1-3.6 and 4]"

      Delete
  92. I mean of course the addition of "housefly eggs" in independent claim 1.

    ReplyDelete
  93. Dear Roel,

    firstly I wish to say I am honoured and happy that you find time to answer my questions.

    Secondly, I must admit that sometimes the sentences in the decisions of DBA are Greek to me, as I am not a native speaker. Nevertheless, I understand that DBA provides an instruction to the Examination Board only in cases which are prima facie wrong, like it was this year the deletion of all points due to a wrong Art. 123(2) amendment.

    I do though believe, that DBA could also investigate whether the amendment "or housefly eggs" would have been derivable from the original disclosure.

    The word "Derivable" is explicitly used in the GL Part H, “Content of the application as originally filed”. The GL do not talk about "what can the skilled person read word for word in the original disclosure". The original disclosure rather includes what the skilled person can "DERIVE" from it.

    In the link below we have the container of claim 1 of Paper B:

    https://wurmkiste.at/produkt/familienwurmkistelaerche/?attribute_variante=Selbstbauset&gclid=EAIaIQobChMIs6ue8pO09gIVFo9oCR1GegFuEAQYAiABEgKix_D_BwE

    Can one say that the skilled man working with these containers every day (routine work in a factory for instance!) would not derive that he can put organic refuse populated with housefly eggs instead of earthworms in it ?

    My question is at least tricky.
    Paper B should have mentioned some staff in view of a container populated with housefly eggs in order to make it unsuitable for earthworms.
    For instance "it is known from the prior art to populate organic refuse with housefly eggs , however the corresponding containers need to have hermetic walls otherwise the housefly would die."

    If we agree that a skilled man can not DERIVE anything then why do we need a skilled man in the EPC?

    Some more transparency regarding the marking of this part of the exam would have been very welcome.






    ReplyDelete
    Replies
    1. "what the skilled person can derive" is not what he may all be able to red into it or to get from it
      The test is "derive/understand directly and unambigously".
      E.g., from "in a range of 1-10", the skilled person does NOT derive "5" - even though it is in 1-10. If he would, a selection invention or a later subrange could never be novel!
      Indeed, no literal correspondence is required, but still only explicit and implicit disclosures that derive directly and unambigously can be derived by the skilled person within 123(2).
      Implicit also does not mean that the tech teaching is changed: it just emans the direct consequence of what is explicitly disclosed. E.g., "the animal resulting from a caterpillar getting mature" is the same as a "butterfly" (simplified example), but is is NOT a disclosure of "an insect", even though it IS a specific type of the generic technical concept "insect".

      Delete
  94. Yesterday, a new successful appeal came online: D 0048/21 () of 30.3.2022
    https://www.epo.org/law-practice/case-law-appeals/recent/d210048du1.html

    D 0048/21 () of 30.3.2022
    Entscheidungsgründe
    1. Die Entscheidung konnte ohne mündliche Verhandlung im schriftlichen Verfahren ergehen, da dem Hauptantrag (s.o. VII.A.) entsprochen wurde.
    […]
    Doppelbestrafung, Gebot der Verhältnismäßigkeit

    5. Die Kammer stellt fest, dass der Prüferbericht die Möglichkeit einer Doppelbestrafung nach den in den Entscheidungen D 13/17 und D 16/17 entwickelten Grundsätzen nicht ausschließt. Die Kammer schließt sich insoweit den in der Entscheidung D 22/21 (Punkte 8. und 9. der Gründe) genannten Erwägungen an und macht sie zum Gegenstand der vorliegenden Entscheidung.

    6. Die Kammer gibt jedoch zu Bedenken, dass Bewerber nicht unter allen Umständen die Zuerkennung der vollen Anzahl von Punkten für gewisse Teillösungen, wie hier z.B. die vom Beschwerdeführer vorgenommene Entfernung der unteren Kammer, erwarten können, auch wenn diese für sich zutreffend sind. Nach Regel 24 Ausführungsbestimmungen zu den Vorschriften über die europäische Eignungsprüfung wird von den Bewerbern erwartet, auf alle in dem Prüfungsbescheid erhobenen Beanstandungen zu antworten und geänderte Ansprüche vorzuschlagen, die alle Erfordernisse des EPÜ erfüllen. Deshalb kann die Lösung der Aufgabe B nicht auf das einfache Sammeln von Punkten für einzelne, aus dem Prüferbericht ableitbare Lösungselemente reduziert werden. Verlangt wird, dass die Gesamtheit der Ansprüche und der entsprechenden Argumente eine vollständige und einheitliche Lösung darstellen (vgl. auch D 19/21, Punkt 13. der Gründe).

    7. Die Kammer erkennt jedoch an, dass die Besonderheit der Bewertungsgrundlage, dass alle Punkte für einen Anspruch und für Argumente betreffend Änderungen aufgrund eines einzigen Fehlers verloren gehen können, für unfair und unverhältnismäßig gehalten werden können, weil die Bewertungsgrundlage möglicherweise nicht genügend zwischen den konkreten Lösungen der jeweiligen Bewerber unterscheidet. Aus diesen Gründen hat die Kammer Zweifel, dass das Gebot der Verhältnismäßigkeit in der vorliegenden Bewertungsgrundlage hinreichend beachtet worden ist. Insoweit verweist die Kammer auf die Entscheidung D 22/21 (Punkte 11. und 12. der Gründe) und stimmt den dortigen Ausführungen auch für die vorliegende Entscheidung ausdrücklich zu.

    8. Im Hinblick auf die genannten Umstände kann die Kammer nicht zu der Schlussfolgerung gelangen, dass die Bewertung der unabhängigen Ansprüche hinsichtlich Neuheit und erfinderische Tätigkeit sowie die Bewertung der Änderung hinsichtlich des Merkmals "Stubenfliegeneier" (siehe Anspruch 5 des Beschwerdeführers) in jeder Hinsicht als gerecht angesehen werden kann. Der angefochtenen Entscheidung kann unmittelbar entnommen werden, dass der Beschwerdeführer von den entsprechenden Prüfern 3/3 Punkte für die unabhängigen Ansprüche, 2/2 für die abhängigen Ansprüche und 3/3 Punkte für die Argumente betreffend die Änderungen erhalten hat. Eine kursorische überprüfung der vom Beschwerdeführer vorgeschlagenen Ansprüche bestätigen, dass seine Antwort zutreffende Lösungselemente beinhalten. Da eine mögliche ungerechte Bewertung nicht ausgeschlossen werden kann, hält die Kammer, unter Berücksichtigung der bestehenden Zweifel zugunsten des Beschwerdeführers, die Beschwerde und folglich die Aufhebung der angefochtenen Entscheidung insoweit für hinreichend begründet.

    ReplyDelete
    Replies
    1. D 0048/21 () of 30.3.2022 [continued]

      Benachteiligung aufgrund eines schweren und eindeutigen Fehlers in der deutschen Fassung der Prüfungsaufgabe B 2021 in Form einer Verletzung des Gleichbehandlungsgebots aufgrund Nichtberücksichtigung einer fehlenden Markierung einer Anspruchsänderung

      9. Der Beschwerdeführer hat schlüssig dargelegt, dass in der deutschen Aufgabenstellung in Anspruch 5 das Merkmal "durch Besprühen des Abfalls mit Wasser" fälschlicherweise nicht als durch die Mandantin vorgenommene Änderung fett markiert wurde und dem Beschwerdeführer für die Identifizierung dieses Fehlers ein deutlicher Zeitverlust entstanden ist, welcher zudem durch die fehlende Möglichkeit in der Prüfungssoftware, die Ansprüche auszudrucken, nebeneinander zu legen und zu vergleichen, verstärkt wurde. Ein Bewerber, der mit der französischen Aufgabe arbeitete, in welcher der beschriebene Fehler nicht enthalten war, hatte also im Vergleich zu einem Bewerber, der mit der deutschen Aufgabe arbeitete, einen nicht unerheblichen Zeitvorteil. In der Nichtberücksichtigung dieses Umstands zugunsten des Beschwerdeführers liege ein Verstoß gegen das Gebot der Gleichbehandlung).

      10. Die Beschwerdekammer in Disziplinarangelegenheiten überprüft gemäß ihrer ständigen Praxis die Einhaltung des Gebots der Gleichbehandlung von Bewerbern der Europäischen Eignungsprüfung als höherrangiges Recht (vgl. D 2/95; D 14/95; D 10/97; D 5/99; D 19/04). Ein zu beanstandender Rechtsfehler liegt dann vor, wenn die Prüfungsbedingungen so gewählt sind, dass sie einen Teil der Bewerber benachteiligen, ohne dass hierfür ein sachlicher Grund erkennbar wäre (vgl. D 3/95, Gründe 3; D 10/97, Gründe 4.1; D 11/19, Gründe 8.3.3 a)).

      11. In den zu Punkt 9. oben beschriebenen Umständen für die Bearbeiter der französischen Aufgabenstellung einerseits und für die Bearbeiter der deutschen Aufgabenstellung (und im übrigen auch der englischen Aufgabenstellung, in der der gleiche Fehler enthalten ist) andererseits ist eine Ungleichbehandlung festzustellen, ohne dass hierfür eine objektive Rechtfertigung erkennbar ist. Weder aus dem Prüferbericht noch aus dem Schreiben des Prüfungssekretariats vom 10. August 2021 ist zu erkennen, dass diese unterschiedlichen Umstände bei der Bewertung der Arbeiten der Bewerber der deutschen Fassung, wie die des Beschwerdeführers, berücksichtigt wurden.

      Delete
    2. D 0048/21 () of 30.3.2022 [continued]

      Antrag auf Vergabe der Gesamtnote "nicht bestanden mit Ausgleichsmöglichkeit" und auf Feststellung des Bestehens der europäische Eignungsprüfung

      12. Gemäß Artikel 24(4), zweiter Satz, VEP hat die Beschwerdekammer die angefochtene Entscheidung aufzuheben, falls die Beschwerde zulässig und begründet ist. Die Sache wird dann an die Prüfungskommission zurückverwiesen mit der Anordnung, eine neuerliche Bewertung der in Frage stehenden Prüfung vorzunehmen zu lassen (vgl. z.B. die zur EEP 2021 ergangenen Entscheidungen D 18/21, D 29/21, D 39/21, D 22/21). Es stellt sich die Frage, ob vorliegend besondere, gegen eine Zurückverweisung sprechende Gründe im Sinn von Artikel 12 der Ergänzenden Verfahrensordnung der Beschwerdekammer in Disziplinarangelegenheiten vorliegen, die eine Direktentscheidung ausnahmsweise rechtfertigten (vgl. hierzu auch D 14/17).

      13. Die Beschwerdekammer ist zur überzeugung gelangt, dass im vorliegenden Fall solche gegen eine Zurückverweisung sprechende besondere Gründe im Sinn von Artikel 12 der Ergänzenden Verfahrensordnung vorliegen. Dem Beschwerdeführer fehlten für die Gesamtnote "nicht bestanden mit Ausgleichsmöglichkeit" für die Prüfungsaufgabe B, die zugleich für das Bestehen der europäischen Eignungsprüfung ausreichend war, lediglich drei Punkte. Im Hinblick auf den pauschal und mehrfach vorgenommenen Abzug der vollen Punktzahl im Rahmen der oben erörterten Doppelbestrafung bei der Bewertung der Lösungen zu den unabhängigen Ansprüchen und der Argumente zur Änderung der Ansprüche (Punkte 5. bis 8.) sowie der Nichtberücksichtigung der Verletzung des Gleichbehandlungsgebots bei der Punktevergabe (siehe oben Punkte 9. bis 11.) bei der Punktevergabe, ist nach Auffassung der Kammer insgesamt die Zuerkennung von jedenfalls drei zusätzlichen Punkten in jeder Hinsicht gerechtfertigt. Unter diesen Umständen bedurfte es keiner Entscheidung der Beschwerdekammer über die Wertigkeit der betroffenen Teilaufgaben. Weiterhin beantragte der Beschwerdeführer auch keine Vergabe einer konkreten Punktezahl für seine Prüfungsarbeit, was eine eingehende fachliche Bewertung der Prüfungsleistung erforderlich gemacht hätte. Er verfolgte auch keinen Antrag auf Vergabe der Gesamtnote "bestanden", der eine Nachprüfung der Beschwerdekammer erforderlich gemacht hätte (vgl. hierzu auch D 14/17).

      14. Folglich hatte die Beschwerdekammer im Ergebnis lediglich zu prüfen, ob eine Bewertung der oben erörterten relevanten Beschwerdegründe des Beschwerdeführers mit weniger als drei Punkten eine unangemessene Ausübung des Ermessens- bzw. Beurteilungsspielraums des zuständigen Prüfungsausschusses wäre. Dies konnte die Beschwerdekammer in Anbetracht der hohen Wertigkeit der genannten Aufgabenteile und der geringen in Frage stehenden Punktezahl ohne Zweifel bejahen.

      15. Die Beschwerdekammer kommt daher zum Schluss, dass besondere, gegen eine Zurückverweisung sprechende Gründe im Sinn von Artikel 12 der Ergänzenden Verfahrensordnung der Beschwerdekammer in Disziplinarangelegenheiten gegeben waren. Dem Hauptantrag, der Prüfungsaufgabe B der Europäischen Eignungsprüfung 2021 des Beschwerdeführers zumindest die Gesamtnote "nicht bestanden mit Ausgleichsmöglichkeit" zu vergeben und darauf gestützt das Bestehen der Europäischen Eignungsprüfung durch den Beschwerdeführer festzustellen, konnte stattgegeben werden (Artikel 24(4), erster Satz, VEP i.V.m. Artikel 22(3) VDV i.V.m. Artikel 111(1), 2. Satz, EPÜ).

      16. Da der Beschwerde nach dem Hauptantrag stattzugeben war, entspricht es der Billigkeit, die Beschwerdegebühr in voller Höhe zurückzuzahlen (Artikel 24(4) VEP).

      Delete
    3. D 0048/21 () of 30.3.2022 [continued]

      Entscheidungsformel

      Aus diesen Gründen wird entschieden:

      1. Die angefochtene Entscheidung wird aufgehoben.

      2. Für die Prüfungsaufgabe B der Europäischen Eignungsprüfung 2021 des Beschwerdeführers wird die Gesamtnote "nicht bestanden mit Ausgleichsmöglichkeit" vergeben.

      3. Es wird festgestellt, dass die Bedingungen des Artikels 14(1) VEP erfüllt sind und der Beschwerdeführer die europäische Eignungsprüfung bestanden hat.

      4. Die Rückzahlung der Beschwerdegebühr wird angeordnet.

      Comment: rather than remitting to the Examination Board and/or B Committee, the DBA itself decided to award a COMPESNABLE FAIL.

      Full text:
      https://www.epo.org/law-practice/case-law-appeals/recent/d210048du1.html

      Delete
  95. D 0026/21 () of 26.4.2022 (as far as relating to B 2021)

    6. Antrag (3): Neuvornahme der Bewertung für Prüfungsaufgabe B

    6.1 Analog zu Antrag (1) legt die Kammer auch den Antrag (3) ("Unter Berücksichtigung besonderer Umstände sei die Bewertung der Bearbeitung der Aufgabe B der EEP des Beschwerdeführers aufgrund eines Zeitverlusts von etwa 15 % (ca. 30 min) verursacht durch technische Störungen des Programms WISEflow sowie Fehlverhaltens des Aufsichtspersonals bei der Störungsbehebung erneut vorzunehmen") so aus, dass die Neuvornahme der Bewertung für die Prüfungsaufgabe B nicht durch die Kammer, sondern nach Zurückverweisung der Angelegenheit durch die Prüfungskommission begehrt wird.
    6.2 Hinsichtlich des vom Beschwerdeführer in den Antrag aufgenommenen Grunds für die begehrte Neubewertung ("aufgrund eines Zeitverlusts von etwa 15 % (ca. 30 min) verursacht durch technische Störungen des Programms WISEflow sowie Fehlverhaltens des Aufsichtspersonals bei der Störungsbehebung") weist die Kammer darauf hin, dass Anträge nur definieren sollen, welche Rechtsfolge begehrt wird, das heißt, welche Rechte in Abänderung der angefochtenen Entscheidung zugesprochen werden sollen. Sie sollen so präzise formuliert werden, dass sie im Falle einer Gutheißung als Entscheidungsformel den Tenor der Entscheidung bilden können. Es geht in Anträgen mithin nur um die von der Kammer auszusprechende Rechtsfolge, nicht um deren Begründung. Begründungen sind in Anträgen nicht aufzunehmen, da unterschiedliche Begründungen in verschiedenen Anträgen für dieselbe Rechtsfolge keinen Unterschied im Ergebnis bewirken. Eine beschwerdeführende Person kann damit nicht erreichen, dass sich die Beschwerdekammer mit unterschiedlichen Begründungen der beschwerdeführenden Person in einer gewissen Reihenfolge auseinandersetzt. Die Begründung ist Sache der Kammer, die nicht an die Rechtsauffassung der beschwerdeführenden Person gebunden ist (vgl. D 11/19, Gründe 1.2). In Anwendung dieser Grundsätze stellt die Kammer daher fest, dass es dem Beschwerdeführer ausschließlich um die Rechtsfolge der Neuvornahme der Bewertung der Prüfungsaufgabe B unabhängig von der Begründung hierfür geht. Entsprechend würde nur die Rechtsfolge und nicht die Begründung in den Tenor der Entscheidung aufgenommen werden, würde dem Antrag stattgegeben.

    6.3 Auch hinsichtlich der Prüfungsaufgabe B kommt eine Verletzung von Regel 19 (3) und (4) ABVEP in Betracht. Der Beschwerdeführer hat hier als Rüge im Sinne von Regel 19 (3) ABVEP schwerwiegende Beeinträchtigungen des Prüfungsablaufs während der Prüfungsaufgabe B aufgrund Funktionsstörungen in der verwendeten Prüfungssoftware WISEflow geltend gemacht, die zu einem deutlichen Zeitverlust geführt hätten. Die Rüge war auch form- und fristgemäß. Die Prüfungskommission hat auch hier in Verletzung ihrer Verpflichtung aus Regel 19 (4) ABVEP keine schriftliche und mit Gründen versehene Entscheidung unter Berücksichtigung aller Beweismittel getroffen. Antrag 3 ist daher ebenfalls begründet. Hinsichtlich der Konsequenzen hieraus gilt das oben in Ziffer 4.5 Gesagte analog.

    ReplyDelete
    Replies
    1. D 0026/21 [continued]:

      6.4 Der Beschwerdeführer hat darüber hinaus auch die inhaltliche Bewertung seiner Bearbeitung der Prüfungsaufgabe B gerügt und vorgetragen, dass seine Aufgabenbearbeitung unzureichend berücksichtigt worden sei. Um auch aus diesem Grund eine Neubewertung durch die Prüfungskommission zu erreichen, hätte der Beschwerdeführer aber substantiiert vortragen müssen, inwiefern bei der Bewertung seiner Bearbeitung ein schwerer und eindeutiger Fehler vorgelegen hat. Die bloßen Behauptungen, dass "Anspruch 1 [] neu und erfinderisch [sei], obgleich dieser Lösungsvorschlag nicht in der abschließenden Auflistung im Prüferbericht aufgeführt [sei]", und dass "[k]eine der Entgegenhaltungen [] den Gegenstand in Anspruch 1 [offenbarten] oder [] diesen [nahelegten]", genügen jedoch in keiner Weise den Anforderungen an einen substantiierten Sachvortrag. Gleiches gilt für die Ausführungen des Beschwerdeführers in Randziffern 34 und 35 seiner Beschwerdebegründung. Auf diese Umstände hat die Kammer den Beschwerdeführer in ihrem Bescheid vom 18. Februar 2022 hingewiesen, ohne dass er hierauf reagiert hat.

      7. Antrag (4): Hilfsweise Rückerstattung der Prüfungsgebühr für Prüfungsaufgabe B und Nichtwertung des Versuchs

      Da Antrag (3) nach Auffassung der Kammer begründet ist, muss sie zum hilfsweise gestellten Antrag (4) ("Hilfsweise zu Antrag (3) sei die Prüfungsgebühr für Teil B der EEP dem Beschwerdeführer zurückzuerstatten und dieser Versuch nicht zu werten, so dass eine erneute Anmeldung für Teil B der EEP nicht als Wiederholung im Sinne von Regel 8 lit. a ff. ABVEP gilt") nicht mehr Stellung nehmen. Inhaltlich wäre es dabei - wie beim hilfsweise gestellten Antrag (2) - ohnehin nur um eine alternative, durch die Prüfungskommission nach Zurückverweisung der Angelegenheit vorzunehmende Reaktion auf die Beeinträchtigungen während des Prüfungsverlaufs gegangen.

      Entscheidungsformel

      Aus diesen Gründen wird entschieden:

      1. Die angefochtene Entscheidung der Prüfungskommission wird aufgehoben.

      2. Die Angelegenheit wird hinsichtlich der Prüfungsaufgaben D1-2 und B zur erneuten Entscheidung an die Prüfungskommission zurückverwiesen.

      3. Die Rückzahlung der Beschwerdegebühr wird angeordnet.

      https://www.epo.org/law-practice/case-law-appeals/recent/d210026du1.html

      Delete
  96. Today, another successful B 2021 appeal was published on the Recent decisions page:
    D 0054/21 () of 25.4.2022
    (full decision: https://www.epo.org/law-practice/case-law-appeals/recent/d210054eu1.html)

    Summary of Facts and Submissions
    I. An appeal was filed against the decision of the Examination Board for the European qualifying examination (EQE) 2021 as communicated to the appellant by letter dated 21 June 2021, which was that he had failed the EQE. On his answer papers he was awarded the following marks: B: 49 COMPENSABLE FAIL, C: 40 FAIL.
    II. By letter dated 25 June 2021, received on 28 June 2021 in the EPO, the appellant filed a notice of appeal including a statement setting out the grounds for appeal regarding both Paper B and Paper C.
    III. With decision dated 11 August 2021 the Examination Board decided to allow the appellant's appeal of 28 June 2021 by awarding the following marks: A: 49 (2019), B: 50 (2021), C: 50 (2021), D: 45 (2019).
    IV. With letter dated 30 August 2021 the appellant filed an appeal including a statement setting out the grounds for appeal against the decision of the Examination Board dated 11 August 2021 awarding him 50 marks for Paper B. With letter dated 31 August 2021 the appellant filed further arguments.
    V. By letter of 20 September 2021, the Examination Secretariat informed the appellant that his appeal had not been allowed by the Examination Board and, consequently, had been submitted to the Disciplinary Board of Appeal (hereinafter "DBA").
    VI. With letters dated 26 September 2021, 10 February 2022 and 29 March 2022 the appellant submitted further arguments and comments in order to support his appeal.

    VII. - VIII. […]

    ReplyDelete
    Replies
    1. D 0054/21 () of 25.4.2022 [continued]

      IX. Furthermore, some of the arguments and allegations put forward and considered as relevant by the appellant can be summarised as follows:
      (a) The appellant alleged at length that Paper B (2021) did not have "the same syllabus and character as before". In Paper B examinations of 2014 to 2019 the candidates were much less penalized by reduction of marks. His answer to Paper B 2021 would have scored many more marks if any marking schemes of the previous twenty years would have applied.
      (b) The appeal dated 25 June 2021 specified that a maximum of 38 marks could possibly not have been awarded despite the fact that his answers were either almost identical or even more detailed when compared to the model solution.
      X. The appellant requests that
      the decision under appeal is set aside,
      the answers to Paper B are remarked in view of the grounds described above,
      the answer paper B 2021 is remarked with at least 56 marks,
      the grade "PASS" is awarded for Paper B 2021 with at least 56 marks,
      the appeal fee is reimbursed.
      Oral proceedings are requested in case the Disciplinary Board of appeal intended to dismiss the appeal.

      Delete
    2. D 0054/21 () of 25.4.2022 [continued]

      Reasons for the Decision
      1. – 3. […]

      Unrewarded solution elements
      4. As far as the appellant contends in his various letters filed during the appeal proceedings that his answers were either almost identical or even more detailed when compared to the model solution or would at least be equivalent to the solutions provided in the Examiners' Report the appeal is without success. With regard to each individual contention the Board would not only have to perform a detailed technical analysis of the facts and features in the opposed patent and their relevance in respect of the respective documents representing the relevant state of the art according to the appellant, as well as of the communication pursuant to Article 94(3) EPC, the client's letter and the third party observation. The Board would also have to evaluate whether the appellant's arguments and proposals deviating from those in the Examiners' Report could be regarded as equivalent technical and reasonable solution and equally valid for the assessment of the aforementioned requirements under the EPC which had then to be awarded a certain amount of marks as requested by the appellant. However, it is not the function of the DBA to reconsider the entire examination procedure on the merits, what would be necessary in order to evaluate and conclude whether a sufficient amount of marks were to be awarded for a "PASS" grade or whether a definite amount of marks could be awarded in view of Rule 6(4)c) IPREE. Thus, examining these arguments put forward by the appellant would clearly be an exercise which could not be done without making a value judgement, which is not a function of the DBA.
      5. In this context the Board notes that candidates cannot expect that they will be awarded the full marks for certain partial solutions under any circumstances, even if these are doubtless correct on their own. According to Rule 24 IPREE, in the Paper B part of the EQE candidates are expected to respond to all points raised in the official communication and to provide amended claims that meet the requirements of the EPC, i.e. all requirements of the EPC. Thus answering Paper B cannot be reduced to the simple exercise of collecting marks for certain solution elements that are derivable from the Examiners' Report. It is also required that the totality of the claims and the corresponding arguments constitute a complete and in itself consistent solution.

      Delete
    3. D 0054/21 () of 25.4.2022 [continued]

      Paper B in previous examinations
      6. A comparison of Paper B 2021 with Papers B in previous EQE cannot be regarded as a suitable and reasonable basis for the evaluation of the allowability of an appeal filed against the awarding of marks regarding Paper B 2021. In this respect it is irrelevant for the present appeal whether Paper B 2021 did have "the same syllabus and character as before" or whether in Paper B examinations of 2014 to 2019 the candidates were much less penalized by reduction of marks. Rather, each annual examination, e.g. Paper B, has to be regarded and assessed separately and on its own, since the technical and legal facts as well as the factual circumstances and conditions underlying the EQE vary each year and hence do not allow a valid and reasonable comparison. However, what is decisive in this regard is, that the conditions were equal for all candidates taking part in the EQE 2021, so that no candidate was disadvantaged in comparison to other candidates in the examination 2021 by the scope, conditions underlying the EQE and the marking system applied for Paper B 2021. The difficulty, the extent of documents and the technical conditions underlying the examination as well as the marking scheme in the EQE in previous years do not affect or influence the assessment, in particular the awarding of marks of Paper B 2021, even if assuming that the previous Paper B examinations actually did not have "the same syllabus and character" compared to Paper B 2021 as alleged by the appellant.

      Delete
    4. D 0054/21 () of 25.4.2022 [continued]

      Potential effects of the "housefly eggs" amendment
      7.-8. […]
      Double penalty was not excluded by the marking scheme
      9. The Examiners' Report does not draw the attention of the Examiners to the problem of double penalty (unlike the Examiners' Report for Paper B 2019), and even less is there any explicit instruction to avoid it. [..]
      10. […] In view of these details the Board finds it plausible that the total marks awarded were decisively influenced by the marking of the "housefly eggs" amendment, without directly accepting the argued amount of the marks possibly lost due to the "housefly eggs" amendment. Given that a possible unfair marking could not be excluded by the Board, the Board finds, giving the appellant the benefit of the doubt, that the appeal is well founded and allowable so that the contested decision is to be set aside according to Article 24(4), second sentence, REE. The Board holds that a new evaluation of the appellant's answer paper B is justified.

      Delete
    5. D 0054/21 () of 25.4.2022 [continued]

      Erroneous paper and unequal treatment
      11. It is settled case law of the DBA that equal treatment of candidates is an issue which may be the subject of appeals under Article 24(1) REE (see the recent decision D 0008/21, point 10.2 of the Reasons and the cases cited). The principle of equal treatment requires that candidates should take part in the examination under equal conditions. Thus it follows from this principle of equal treatment that unequal conditions which may cause unjustified disadvantages for candidates should be compensated, to the extent feasible. It is an undisputed fact that there had been a mark-up error in the examination paper. This is apparent when comparing the English and French versions of the Paper B 2021, page 24. The amended claim 5 as suggested by the client contains an amendment as compared with the originally filed claims. The client inserted the feature "by spraying water on said refuse", among other amendments made. All amendments made by the client were highlighted with bold, except this amendment. The error meant that an instruction of the client was easily overlooked, namely the addition of the feature "by spraying water on said refuse". While it may not be quite clear from the Examiners' Report how the leaving out this feature may have been penalised exactly by the marking scheme, it is clear that the feature was an expected feature (point 3.2 of the Examiners' Report), so it is safe to assume that marks were deducted if the feature was missing.
      12. The Board finds the appellant's arguments plausible that this error was difficult to discover under the circumstances of the online examination, and even if discovered, the candidates were faced with a confusing set of facts. On this basis, the Board accepts that this difference between the French and the English version may have had a significant impact on the answer paper of a candidate, and in this manner candidates writing the French and English versions had to write the Paper B 2021 under different conditions. The Board considers that these different conditions amount to an unequal treatment of the candidates. Such unequal treatment deserves some form of compensation, but such is not apparent from the Examiners' Report or the marking of the candidate. In this regard the Board refers to decision D 0008/21, points 10.1 to 10.3 and point 12.1 of the Reasons. The Board explicitly endorses these reasons of D 0008/21 and agrees with the conclusion stated in point 12.1 that the unequal treatment must be compensated.

      Delete
    6. D 0054/21 () of 25.4.2022 [continued]


      Request for re-marking and award of grade
      13. The appellant further requests that his answer paper be awarded a PASS with at least 56 marks. With the award of 56 marks for Paper B the appellant would have satisfied all of the conditions for passing the examination according to Rule 6(4)a), b) and c) IPREE. The Board follows the case law of the DBA (see e.g. decisions D 0024/17, point 15 of the Reasons, D 0013/17 and D 0016/17, point 4 of the Reasons) and considers that the Board itself in principle cannot perform the re-marking requested. An assessment of the appellant's answer paper for determining the marks to be awarded would be equivalent to a review of the marking on the merits and thus would require value judgments which, according to the established jurisprudence (following D 1/92, OJ EPO 1993, 357), falls outside the competence of the Board, as already stated above in points 3. And 4. Therefore, the Board decides to remit the case to the Examination Board with the order to instruct the competent Examination Committee to undertake a new marking of the appellant's Paper B of the European qualifying examination 2021 under its powers pursuant to Article 6(5), last sentence, REE and to award a grade to the appellant on the basis of the re-marking. The re-evaluation should be based on a marking scheme which is fairly proportional and avoids multiple penalties, according to the principles as set out in decision D 0022/21, point 16 of the Reasons. These principles are also supported by the present Board.
      Reimbursement of the appeal fee
      14. The appellant requested reimbursement of the appeal fee. If the Board of Appeal allows the appeal, it orders reimbursement in full or in part of the appeal fee if this is equitable in the circumstances of the case (Article 24(4), third sentence, REE). Given that the present appeal is successful, a full reimbursement of the appeal fee is equitable. Therefore, the appeal fee is to be reimbursed in full.

      Delete
    7. D 0054/21 () of 25.4.2022 [continued]

      Order

      For these reasons it is decided that:

      1. The decision under appeal is set aside.
      2. The case is remitted to the Examination Board with the following order:
      a) the Examination Board is instructed to determine the applicable compensation for the answer paper of the appellant for the Paper B of the European qualifying examination 2021 in respect of
      (i) the unequal treatment caused by the erroneous examination paper (mark-up error),
      (ii) the double penalty as set out in the reasons above and
      b) to instruct the competent Examination Committee to perform a re-marking of the appellant's answer paper for the Paper B, also taking into account the determined compensation, and
      c) to award a grade to the answer paper under Rule 6(3) and (4) IPREE accordingly.
      3. The appeal fee is to be reimbursed in full.

      Delete
    8. Roel, are you aware of any A exam 2021 appeals? It seems that B exam had a lot of appeals, most of which successfus as it seems. Even today, some people shared on LinkedIn that they passed 2021 beacause of successful B appeal. Are there any appeals for A ?
      /By the way the number of appeals shows the good quality level of the paper/

      Delete
    9. I do not know whether any A 2021 appeals were filed. None have been published yet. Maybe still pending? Maybe they were filed, but none of them were successful and the candidates withdrew their appeals before formal decision was taken (e.g., after non-successful interlocutory revision, or after a preliminary opinion of the dBA) so as to get the appeal fee back in part or in full - those appeals do not become published.

      Delete
    10. Thank you. Maybe there are none or there are unsuccessful because A paper was more straightforward. Apart from last year's D, who could believe a year ago that so many marks could be awarded (14); I thought that the practice was to file an apeeal for a few missing marks (2-4). I passed B but now I wonder shall I feel a bit sorry that did not file an appeal for A; obviously it is not impossible to gain many mark

      Delete
  97. Good Morning. Does anyone know how many marks were received in the end by the appellant of D22/21 ? did he/she in the end manage 45 or 50 from initial 37 ? or maybe more (marks)? thanks for any info.

    ReplyDelete
    Replies
    1. Don't know if this is the case number but today a woman shared on Linkedin that passed B 2021 because of awarded 14 additional marks , so the starting point must have been 36-37 or even lower

      Delete
  98. or in D19/21, same reason "proportionality"/"housefly eggs" ?

    ReplyDelete

Post a Comment

Oldest Older 201 – 400 of 421 comments Newer Newest